Nghiên cứu định lý Viète và ứng dụng – Nguyễn Thành Nhân

Tài liệu gồm 56 trang, được biên soạn bởi tác giả Nguyễn Thành Nhân, khai thác chuyên sâu định lý Viète và ứng dụng.

A. LỊCH SỬ.
B. ĐỊNH LÝ VIÈTE.
Trong toán học, định lý Viète hay công thức Viète (có khi viết theo phiên âm tiếng Việt là Vi-ét), do nhà toán học Pháp François Viète tìm ra, nêu lên mối quan hệ giữa các nghiệm của một phương trình đa thức (trong trường số phức) và các hệ số của nó.

October 4, 2021
[NGUYN THÀNH NHÂN TRƯỜNG ĐI HC AN GIANG]
1
Nghiên cu đnh lý Viète và ng dng | mathvn.com
NG DỤNG ĐỊNH LÝ VIÈTE
ĐẠI S SƠ CẤP
A. LCH S.
François Viète, Seigneur de la Bigotière ( tiếng Latinh : Franciscus Vieta ; 1940 - 23
tháng 2 năm 1603) là mt nhà toán hc ngưi Pháp có công trình v đại s mi là mt
bước tiến quan trọng đi vi đi s hiện đại, do vic s dng sáng to các ch cái làm
tham s trong phương trình và đồng thi ng dng chúng trong vic biến đi và gii
phương trình. Ông là mt luật sư về thương mi, và tng là y viên hi đng bí mt cho
c Henry III và Henry IV ca Pháp.
Ông đã phát hin ra mi liên h gia các nghim và các h s của phương trình. Ông còn
là mt chuyên gia v gii các mt mã trong thế chiến gia Pháp và Tây Ban Nha.
Ông mất năm 1603.
Thành tu ni bt: Đại số mới.
Nền
Vào cuối thế kỷ 16, toán học được đặt dưới sự bảo trợ kép của người Hy Lạp, họ đã
mượn các công cụ của hình học và người Ả Rập, những người cung cấp các thủ tục cho
phép giải. Vào thời của Viète, đại số do đó dao động giữa số học, điều này làm xuất hiện
một danh sách các quy tắc và hình học có vẻ chặt chẽ hơn.
Đại số biểu tượng của Viète
Viète đã to ra nhiều đi mi: công thc nh thc , s được Pascal và Newton ly, và
các h s ca đa thức thành tng và tích các gc ca nó , được gi là công thc Viète .
Đại số hình học
Viète rt thành tho trong hu hết các công c hiện đại, nhm mục đích đơn giản hóa các
phương trình bằng cách thay thế các đại lưng mi có mi liên h nht đnh vi các đi
ợng chưa biết ban đầu. Mt tác phm khác ca ông, Recensio canonica effectionum
learningarum , mang du n hiện đại, sau này được gi là hình học đại s mt b sưu
tp các gii thiu cách xây dng các biu thức đại s ch vi vic s dụng thưc và
compass.
October 4, 2021
[NGUYN THÀNH NHÂN TRƯỜNG ĐI HC AN GIANG]
2
Nghiên cu đnh lý Viète và ng dng | mathvn.com
B. ĐỊNH LÝ VIÈTE.
Trong toán hc, định lý Viète hay công thc Viète (có khi viết theo phiên âm tiếng
Vit Vi-ét), do nhà toán hc Pháp François Viète tìm ra, nêu lên mi quan h gia
các nghim ca mt phương trình đa thức (trong trưng s phc) và các h s ca nó.
I. Định lý Viète cho phương trình bc hai.
1. Bài toán m đầu.
Xét phương trình bậc hai:
2
01y ax bx c a
.
Gi s:
2
40b ac
.
Ta có:
1
2
2
2
b
x
a
b
x
a
là hai nghim tng quát của phương trình
2
0ax bx c
.
Khi đó:
.
2. Định lý Viète.
Extra Techniques
Định lý Viète
Nếu
12
,xx
là hai nghiệm (trên trường s phc , có th nghiệm đơn hoặc nghim kép)
của phương trình:
2
0ax bx c
, thì:
12
12
b
x x S
a
c
x x P
a

.
Chng minh:
October 4, 2021
[NGUYN THÀNH NHÂN TRƯỜNG ĐI HC AN GIANG]
3
Nghiên cu đnh lý Viète và ng dng | mathvn.com
Gi s:
12
,xx
là hai nghim của phương trình
2
0ax bx c
.
Khi đó, phương trình bc hai
1
tương đương với phương trình
12
y a x x x x
.
Như vậy, ta có đẳng thc:
2
12
ax bx c a x x x x
.
Hay:
22
1 2 1 2
ax bx c ax a x x x ax x
.
Đồng nht h s hai vế, ta thu được:
12
12
12
12
b
xx
b a x x
a
c
c ax x
xx
a


(đpcm).
Như vậy, mt câu hỏi đưc đt ra: Liu rng có hay không mt Định Viète tng quát
trên trưng s thc cho mt đa thc có bc
n
?
Câu tr lời là có và xin được trình bày tiếp phần dưới đây.
II. Định lý Viète cho phương trình đa thc bt k.
1. Bài toán m đầu.
Xét phương trình bậc
n
theo n
x
tổng quát như sau:
1
1 1 0
... , 0 2
nn
n n n
y a x a x a x a a
Gi s:
, 1,
i
x i n
n
nghim của phương trình
1
1 1 0
... 0
nn
nn
a x a x a x a
.
Khi đó, phương trình bc
n
tương đương với phương trình:
12
...
nn
y a x x x x x x
Như vậy, ta có:
1
1 1 0 1 2
... ...
nn
n n n n
a x a x a x a a x x x x x x
1
1 1 0
12
...
...
n n n
n n n
nn
n elements
a x a x a x a a x
a x x x



October 4, 2021
[NGUYN THÀNH NHÂN TRƯỜNG ĐI HC AN GIANG]
4
Nghiên cu đnh lý Viète và ng dng | mathvn.com
1 2 1 3 1 1
1
2
1
1 2 1 1 2 2 2 3
... ...
.......................................................................
1 ... ... ... ...
n k k n n
nn
elements
n
n n n n n
n element
a x x x x x x x x
a x x x x x x x x x x







12
1 ...
s
n
nn
a x x x




Đồng nht h s hai vế, ta thu được Định lý Viète m rng như sau:
Extra Techniques
Định lý Viète m rng
Nếu
, 1, ,
i
x i n n

là hai nghiệm (trên trường s phc , có th nghiệm đơn hoặc
nghim kép) của phương trình:
1
1 1 0
... 0
nn
nn
a x a x a x a
, thì:
1 2 1
1 2 1
1
12
1
2
1 2 1 3 2 3 1
1
1 2 1 1 2 2
1 ...
...
....
............................................................
... ... ... ...
n
n
n
n
in
i
n
n
n
i j n n
i j n
n
n
i i i n n n
i i i n
a
x x x x
a
a
x x x x x x x x x x
a
x x x x x x x x x x

1
1
23
0
12
1
... 1 .
... 1 .
n
n
n
n
n
in
i
n
a
x x x
a
a
x x x x
a

.
Lưu ý: Trong mi hàng
k
bt k, vế trái của đẳng thc là tng ca các tích tng cm
k
các nghim của phương trình trên. Và vế phi ca đng thức được tính mt cách tng
quát theo công thc:
1
k
nk
n
a
a
.
October 4, 2021
[NGUYN THÀNH NHÂN TRƯỜNG ĐI HC AN GIANG]
5
Nghiên cu đnh lý Viète và ng dng | mathvn.com
Mt s tổng quát thưng gp:
Phương trình bậc ba:
Nếu
1 2 3
,,x x x
là nghim của phương trình:
32
0ax bx cx d
thì công thc Viète
cho ta:
1 2 3
1 2 2 3 3 1
1 2 3
b
x x x
a
c
x x x x x x
a
d
x x x
a

.
H qu 1 của định lý Viète:
Gi s phương trình:
1
1 1 0
... 0 3
nn
nn
a x a x a x a
có các h s
, 0,
i
a i n
tha mãn:
1
22
2 2 1
01
0
nn
kk
kk
i a a



khi và ch khi:
1x
là mt nghim của phương trình
3
.
1
22
2 2 1
01
0
nn
kk
kk
ii a a



khi và ch khi:
1x 
là mt nghim của phương trình
3
.
Chng minh
i
:
Gi s:
1
22
2 2 1 0 1 1
01
0 ...
nn
k k n n
kk
a a a a a a



.
Khi đó:
1
1 1 1 1
3 ... 0
nn
n n n n
a x a a x a a x a

;
1
11
1 2 2 3
1 2 1
1 1 ... 1 0
1 ... 1 ... 1 ... 1 0
nn
nn
n n n n
nn
a x a x a x
x a x x a x x a x a


October 4, 2021
[NGUYN THÀNH NHÂN TRƯỜNG ĐI HC AN GIANG]
6
Nghiên cu đnh lý Viète và ng dng | mathvn.com
1x
là mt nghim.
i
:
Gi s:
1x
là mt nghim của phương trình
3
, khi đó, ta có:
1
22
1
1 1 0 1 1 0 2 2 1
01
1 1 ... 1 0 ... 0
nn
nn
n n n n k k
kk
a a a a a a a a a a


.
Vy:
i
hoàn toàn được chng minh.
ii
:
Bài toán ph:
Ta có:
2 1 2 2 1
21
2
2
2
1 1 ... 1 ... 1 1 . 4
*
1 1 1 1 ... 1 . 5
k
t t t k
t
k k k k
t
x x x x x x
x x x x x

Mt điều đáng nói, ở đây
k
hoc chn hoc lẻ, nhưng điều đó không quan trng vì nếu
2k
thì ta xét
1
k
x
như ở đẳng thc
5
, nếu
21k

thì hoàn toàn rút đưc
ng nhân t
1x
theo đng thc
4
. Như vậy dù
k
là bi
nào đó của
2
, thì đến
mt s đủ lớn các bưc (sau
bước), ta s thu được nhân t
1x
.
Gi s:
11
2 2 2 2
2 2 1 0 2 2 1
0 1 1 1
0
n n n n
k k k k
k k k k
a a a a a


.
Khi đó:
1
12
1 1 2 2 1 1
3 1 1 ... 0
nn
nn
n n n n
a x a a x a a x a a x a

.
1 1 2 1
1 ... 1 0 1
n n n n
x a a a x a x




là mt nghim ca
3
.
ii
: Gi s:
1x 
là mt nghim của phương trình
3
, khi đó, ta có:
October 4, 2021
[NGUYN THÀNH NHÂN TRƯỜNG ĐI HC AN GIANG]
7
Nghiên cu đnh lý Viète và ng dng | mathvn.com
11
1 1 0 1 1 0
1
22
2 2 1
01
1 1 ... 1 0 1 1 ... 1 ... 0
0
n n n n k
n n n n k
nn
kk
kk
a a a a a a a a a
aa




.
Vy:
ii
hoàn toàn được chng minh.
Extra Techniques
Study tips
Xét phương trình:
2
0 0 6ax bx c a
.
Nếu phương trình
6
có tng
0a b c
thì:
1
2
1x
c
x
a
là hai nghim ca
6
.
Nếu phương trình
6
có tng
0abc
thì:
1
2
1x
c
x
a


là hai nghim ca
6
.
H qu 2 của định lý Viète:
Gi s phương trình:
1
1 1 0
... 0 3
nn
nn
a x a x a x a
n
nghim, kí hiu
, 1,
i
x i n
.
Nếu ta đặt:
1 2 1
1 2 1
1
1
2
1
1
1 ...
1
........................
...
n
n
n
i
i
n
ij
i j n
n
n i i i
i i i n
n
ni
i
Sx
S x x
S x x x
Sx
October 4, 2021
[NGUYN THÀNH NHÂN TRƯỜNG ĐI HC AN GIANG]
8
Nghiên cu đnh lý Viète và ng dng | mathvn.com
Khi đó:
, 1,
i
x i n
là nghim của phương trình:
1
12
1 2 1
... 1 . ... 1 1 0 7
k n n
n n n n k
k n n
x S x S x S x S x S
.
Chng minh:
Theo đnh lý Viète m rng, ta suy ra:
Nếu
, 1,
i
x i n
là nghim ca
3
, thì:
1 2 1
1 2 1
1
12
1
2
1 2 1 3 2 3 1
1
1 2 1 1 2 2
1 ...
...
....
............................................................
... ... ... ...
n
n
n
n
in
i
n
n
n
i j n n
i j n
n
n
i i i n n n
i i i n
a
x x x x
a
a
x x x x x x x x x x
a
x x x x x x x x x x

1
1
23
0
12
1
... 1 .
... 1 .
n
n
n
n
n
in
i
n
a
x x x
a
a
x x x x
a

Như vậy, d dàng ta có:
1
1
2
2
1
1
1
0
..................
1.
1.
n
n
n
n
n
n
n
n
n
n
a
S
a
a
S
a
a
S
a
a
S
a



.
Vậy phương trình
7
tương đương với:
12
12
10
... 0
n n n
nn
n n n n
aa
aa
x x x x
a a a a


.
October 4, 2021
[NGUYN THÀNH NHÂN TRƯỜNG ĐI HC AN GIANG]
9
Nghiên cu đnh lý Viète và ng dng | mathvn.com
Vì:
12
deg , ,..., 0
nn
f x x x n a
.
Nhân c hai vế cho
n
a
, ta thu được phương trình
3
(đpcm).
Extra Techniques
Study tips
Xét phương trình:
2
0 0 6ax bx c a
vi
0
, nếu ta đặt:
12
12
S x x
P x x

.
Thì:
12
,xx
là hai nghim của phương trình
22
0, 4x Sx P S P
.
C. MT S TIPS GII NHANH CÁC BÀI TOÁN NG DỤNG ĐỊNH
LÝ VIÈTE.
I. Du nghim của phương trình bc hai.
Du nghim
1
x
2
x
12
S x x
12
P x x
Điu kin cn
Điu kiện đủ
Trái du
?
0
0P
Cùng du
0P
Cùng dương
0, 0PS
Cùng âm
0, 0PS
II. Mt s đẳng thc cần lưu ý.
2
2 2 2
1 2 1 2 1 2
2
3 3 2
1 2 1 2 1 2 1 2
2
2
22
4 4 2 2
1 2 1 2 1 2 1 2
12
1 2 1 2
2
2
2 1 1 2 1 2 2 1
22
3 . 3
2 2 2 2
11
44
i x x x x x x S P
ii x x x x x x x x S S P
iii x x x x x x x x S P P
xx
P
iv
x x x x S
v x x x x x x S P x x






October 4, 2021
[NGUYN THÀNH NHÂN TRƯỜNG ĐI HC AN GIANG]
10
Nghiên cu đnh lý Viète và ng dng | mathvn.com
III. ng dụng đa thức đối xng đ gii quyết các bài tp áp dụng định lý Viète.
1. Định nghĩa. Gi s
A
là một vành giao hoán có đơn vị,
12
, ,...,
n
f x x x
là một đa thức
ca vành
12
, ,...,
n
A x x x


. Đa thức
12
, ,...,
n
f x x x
được gi là mt đa thức đi xng ca
n
n nếu
12
12
, ,..., , ,...,
n
n
f x x x f x x x
vi mi phép thế
1 2 ....
1 2 ....
n
n




12
, ,...,
n
f x x x
suy ra t
12
, ,...,
n
f x x x
bng cách thay trong
12
, ,...,
n
f x x x
,
1
x
bi
1
,....,
n
xx
bi
n
x
.
2. Định lý 1. B phn gồm các đa thức đi xng ca vành
12
, ,...,
n
A x x x


là mt vành
con ca vành
12
, ,...,
n
A x x x


.
Chng minh:
Gi s
12
, ,...,
n
f x x x
12
, ,...,
n
g x x x
là những đa thức đi xng ca vành
12
, ,...,
n
A x x x


, theo định nghĩa ta có:
12
12
, ,..., , ,...,
n
n
f x x x f x x x
12
12
, ,..., , ,...,
n
n
g x x x g x x x
vi mi phép thế
1 2 ....
1 2 ....
n
n




. Thế thì:
1 2 1 2
1 2 1 2
, ,..., , ,..., , ,..., , ,...,
nn
nn
f x x x g x x x f x x x g x x x
,
1 2 1 2
1 2 1 2
, ,..., . , ,..., , ,..., . , ,...,
nn
nn
f x x x g x x x f x x x g x x x
October 4, 2021
[NGUYN THÀNH NHÂN TRƯỜNG ĐI HC AN GIANG]
11
Nghiên cu đnh lý Viète và ng dng | mathvn.com
vi mi phép thế
. T đó, suy ra bộ phn gồm các đa thức đối xng ca vành
12
, ,...,
n
A x x x


là mt vành con ca vành
12
, ,...,
n
A x x x


.
Chú ý: th coi mi phn t ca vành
A
mt đa thức đối xứng đặc bit. Tht vy,
aA
thì ta có th viết:
0 0 0
12
...
n
a x x x
.
Các đa thức đi xứng cơ bản:
1 2 1
12
1 1 2
1
2 1 2 1 3 2 3 1
1
1 1 2 1 1 2 2 2 3
1 ...
...
....
............................................................
... ... ... ... ...
n
n
n
ni
i
n
n n i j
i j n
n n n n n i i i
i i i
x x x x
x x x x x x x x x x
x x x x x x x x x x x x x
1
12
1
...
n
n
n
n n i
i
x x x x

Theo Định 1 thì mọi đa thức của các đa thức đối xứng bản
12
, ,...,
n
cũng
một đa thức đối xng ca
n
n
12
, ,...,
n
x x x
. Chiều ngược lại cũng đúng, đó chính ni
dung ca định lý cơ bản v đa thức đi xng da trên các B đề sau.
3. B đề 1. Gi s
12
, ,...,
n
f x x x
một đa thức đối xng khác
0
1 2 3
1 2 3
...
n
a
a a a
n
x x x x
là hng t cao nht ca nó, thế thì:
12
...
n
a a a
.
Chng minh:
Ta phi chng minh:
1
, 2,
ii
a a i n
. Vì
1
,...,
n
f x x
là một đa thức đi xng nên nếu
thay
1i
x
bi
i
x
và hoán v ngược li thay
i
x
bi
1i
x
, ta được:
1
1
11
... ...
i i n
a a a
a
i i n
x x x x
cũng là mt hng t ca
1
,...,
n
f x x
.
Gi s:
1ii
aa
, khi đó:
October 4, 2021
[NGUYN THÀNH NHÂN TRƯỜNG ĐI HC AN GIANG]
12
Nghiên cu đnh lý Viète và ng dng | mathvn.com
1 2 1 1 2 1
,..., , , ,..., ,..., , , ,...,
i i i n i i i n
a a a a a a a a a a
.
Điu này mâu thun vi gi thuyết
1
1
11
... ...
i i n
a a a
a
i i n
x x x x
là hng t cao nht.
4. B đề 2. Gi s
1
,...,
n
aa
là nhng s t nhiên sao cho:
12
...
n
a a a
thế thì đa thc
12
1 2 2 3
1 2 1 2 1
, ,..., ...
n n n
a a a
a a a a
n n n
f x x x


trong đó
1
,...,
n

là các đa thc đi xứng cơ bản, có hng t cao nht là
12
12
...
n
a
aa
n
x x x
.
Chng minh:
Các hng t cao nht ca
1 2 1
, ,..., ,
nn
theo th t là:
1 1 2 1 2 1 1 2
, ,..., ... , ...
nn
x x x x x x x x x
Ta có hng t cao nht ca
1
,...,
n
f x x
là:
2 3 1
1 2 1 2
1 1 2 1 2 1 1 2 1 2
... ... ... ...
n n n
n
a a a a a
a
a a a a
n n n
x x x x x x x x x x x x

(đpcm).
S dĩ ta kết luận đưc hng t như thế là do ta chứng minh đưc mt Định lý sau:
Định lý
*
. Gi s
12
, ,...,
n
f x x x
12
, ,...,
n
g x x x
là hai đa thức khác không ca vành
12
, ,...,
n
A x x x


có hng t cao nht theo th t
1
11
11
...
n
a
a
n
c x x
1
11
11
...
n
b
b
n
d x x
. Nếu
11
0cd
thì hng t cao nht ca đa thc tích
1 2 1 2
, ,..., . , ,...,
nn
f x x x g x x x
11
11 11
1 1 1
...
nn
ab
ab
n
c d x x
.
Chng minh:
Gi s:
11
11 ln
1 1 n
11
1 2 1 1 1
1 2 1 1 1
, ,..., ... ... ...
, ,..., ... ... ...
nl
n m m
aa
aa
n n l n
b b b
b
n n m n
f x x x c x x c x x
g x x x d x x d x x
đã được sp xếp theo li t đin.
Điều đó có nghĩa là:
October 4, 2021
[NGUYN THÀNH NHÂN TRƯỜNG ĐI HC AN GIANG]
13
Nghiên cu đnh lý Viète và ng dng | mathvn.com
11 1 1
,..., ,..., , 2,
n i in
a a a a i l
11 1 1
,..., ,..., , 2,
n i in
b b b b i m
.
Ta s chng minh:
11
11 11
1 1 1
...
nn
ab
ab
n
c d x x
là hng t cao nht của đa thức tích
11
,..., . ,...,
nn
f x x g x x
.
Nhân
1
,...,
n
f x x
vi
1
,...,
n
g x x
, ta đưc:
11
1 1 1
,
,..., . ,..., ... , 1, ; 1,
i j in jn
a b a b
n n i j n
ij
f x x g x x c d x x i l j m

.
Mi hng t
11
1
...
i j in jn
a b a b
i j n
c d x x

cho ta phn t
11
,...,
n
i j in jn
a b a b
.
Nhưng vì, ta lại có:
+) Nếu
11
,..., ,...,
nn
a a b b
thì
1 1 1 1
,..., ,..., ,
n n n n
a c a c b c b c
1
,...,
n
n
cc
.
Tht vy! Vì:
11
,..., ,...,
nn
a a b b
nên có mt ch s
1,in
sao cho:
1 1 1 1
,..., ,
i i i i
a b a b a b

.
Do đó:
1 1 1 1 1 1 1 1
,..., ,
i i i i i i i i
a c b c a c b c a c b c
(đpcm).
+) Nếu
11
,..., ,...,
nn
a a b b
11
,..., ,...,
nn
c c d d
, thì:
1 1 1 1
,..., ,...,
n n n n
a c a c b d b d
.
Tht vy!
Ta có:
1 1 1 1 1 1
,..., ,..., ,...,
n n n n n n
a c a c b c b c b d b d
(đpcm).
Do vy, ta có các bất đng thc sau:
October 4, 2021
[NGUYN THÀNH NHÂN TRƯỜNG ĐI HC AN GIANG]
14
Nghiên cu đnh lý Viète và ng dng | mathvn.com
11 11 1 1 11 1 1
11 11 1 1 1 11 1
11 11 1 1 1 1
,..., ,..., , 2,
,..., ,..., , 2,
,..., ,..., , 2, , 2,
n n j n jn
n n i in n
n n i j in jn
a b a b a b a b j m
a b a b a b a b i l
a b a b a b a b i l j m
Vy hng t
11
11 11
1 1 1
...
nn
ab
ab
n
c d x x
chính là hng t cao nht của đa thức tích.
5. B đề 3.
Gi s
1
,...,
n
g

là một đa thức ca các đa thc đi xng cơ bản
11
11
1 1 1 1
,..., ... ... ...
n m mn
a a a
a
n n m n
g c c
trong đó
0, 1,
i
c i m
, và
11
,..., ,..., ,
i in j jn
a a a a i j
.
Thế thì:
1
,..., 0
n
g

.
Chng minh:
Trong
1
,...,
n
g

, thay
1
bng
12
... ,...,
nn
x x x
bng
12
...
n
x x x
ta đưc mt đa
thc ca các n
12
, ,...,
n
x x x
.
1 2 1 2 1 1
1
... ,..., ... ,..., ,...,
m
n n n i n
i
g x x x x x x f x x f x x
vi
1
1 1 1 1 2 1 2 3
,..., ... ... ... , 1,
i in
aa
nn
f x x c x x x x x x i m
.
Hng t cao nht của đa thức
11
,...,
n
f x x
theo Định lý
*
là:
12 1
12
11
1 1 1 2 1 2 1 2
... ... ...
n
i i m
aa
k k k
a
n i k
c x x x x x x c x x x
October 4, 2021
[NGUYN THÀNH NHÂN TRƯỜNG ĐI HC AN GIANG]
15
Nghiên cu đnh lý Viète và ng dng | mathvn.com
vi:
1 2 1
22
...
...
.................................
i i in i
i in i
in in
a a a k
a a k
ak

.
Hng t cao nht ca mi đa thc
1
,...,
in
f x x
, cho ta phn t
12
, ,...,
n
i i in
k k k
.
Ta có:
1 2 1 2
, ..., , ,..., ,
i i in j j jn
k k k k k k i j
.
Vì nếu:
1 2 1 2
, ..., , ,..., ,
i i in j j jn
k k k k k k i j
, thì:
1 1 2 1 2 1
2 2 3 2 3 2
.................................................
i i i j j j
i i i j j j
in in jn jn
a k k k k a
a k k k k a
a k k a
vi
ij
, mâu thun vi gi thuyết.
n
sp th t toàn phn nên b phn hu hn gm các phn t
12
, ...,
i i in
k k k
vi
1,im
có phn t ln nht, chng hn
11 12 1
, ,...,
n
k k k
là phn t ln nhất. Do đó
1
11
11
...
n
k
k
n
c x x
là hng
t cao nht ca
1
,...,
n
f x x
.
Vy:
11
,..., ,..., 0
nn
g f x x


(đpcm).
H qu. Gi s:
11
11
1 2 1 1 1
, ,..., ... ... ...
n m mn
a a a
a
n n m n
h x x x c x x c x x
11
11
''
1 2 1 1 1 1
' , ,..., ... ... ...
n m mn
a a a
a
n n m n
h x x x c x x c x x
là hai đa thức trong đó
11
,..., ,...,
i in j jn
a a a a
khi
ij
, sao cho:
October 4, 2021
[NGUYN THÀNH NHÂN TRƯỜNG ĐI HC AN GIANG]
16
Nghiên cu đnh lý Viète và ng dng | mathvn.com
11
,..., ' ,...,
nn
hh
Thế thì
', 1,
ii
c c i m
.
Chng minh:
Gi s
'
ii
cc
.
Đặt:
1 1 1
,..., ,..., ' ,...,
n n n
g h h

.
11
11
1 1 1 1
' ... ... ' ...
n m mn
a a a
a
n m m n
c c c c
Vì:
11
'cc
, nên
11
'0cc
.
Theo B đề 3, ta có:
1
,..., 0
n
g

.
Nhưng theo giả thuyết thì:
1
,..., 0
n
g

, mâu thun.
6. Định lý 2. ịnh lý cơ bản v đa thức đi xng).
Gi s
1 2 1 2
, ,..., , ,...,
nn
f x x x A x x x


là một đa thức đối xứng khác không, khi đó có mt
và ch một đa thức
1 2 1 2
, ,..., , ,...,
nn
h x x x A x x x


sao cho
1 2 1 2
, ,..., , ,...,
nn
f x x x h
.
Trong đó
12
, ,...,
n
là các đa thc đi xứng cơ bản.
Chng minh:
S tn ti.
Ta hãy sp xếp
12
, ,...,
n
f x x x
theo li t điển, gi s
12
12
...
n
a
aa
n
x x x
là hng t cao nht ca
12
, ,...,
n
f x x x
. Theo B đề 1, ta có:
12
...
n
a a a
Mt khác, theo B đề 2, thì đa thức:
October 4, 2021
[NGUYN THÀNH NHÂN TRƯỜNG ĐI HC AN GIANG]
17
Nghiên cu đnh lý Viète và ng dng | mathvn.com
1
1 2 2 3
1 2 1
...
n n n
a a a
a a a a
nn


cũng có hng t cao nht là:
12
12
...
n
a
aa
n
x x x
.
Xét hiu:
1
1 2 2 3
1 1 1 1 2 1
,..., ,..., ...
n n n
a a a
a a a a
n n n n
f x x f x x



.
Nếu
11
,..., 0
n
f x x
, thì ta sp xếp nó theo li t điển và gi s
12
12
...
n
b
bb
n
x x x
là hng t cao nht ca nó.
Theo Định lý 1 thì
1 1 2
, ,...,
n
f x x x
cũng là một đa thức đi xng, và do đó ta có:
12
...
n
b b b
.
Mt khác, t biu thc ca hiệu hai đa thức, ta có:
11
,..., ,...,
nn
a a b b
Do đó:
11
ab
.
Xét hiu:
1
1 2 2 3
2 1 1 1 2 1 2 1
,..., ,..., ...
n n n
b b b
b b b b
n n n
f x x f x x



.
Nếu
21
,..., 0
n
f x x
, ta hãy sp xếp nó theo li t điển và gi s
1
1
...
n
c
c
n
xx
là hng t cao nht của nó. Cũng lý luận tương t đối vi
11
,...,
n
f x x
, ta được:
12
...
n
c c c
Vi
October 4, 2021
[NGUYN THÀNH NHÂN TRƯỜNG ĐI HC AN GIANG]
18
Nghiên cu đnh lý Viète và ng dng | mathvn.com
1 2 1 2
, ,..., , ,...,
nn
b b b c c c
.
Ta nhn thy rng dãy
1 1 1
,..., ,..., ,..., ...
n n n
a a b b c c
không th gim vô hn, tc là
quá trình lp lun trên din ra không th vô tn. Sau mt s hu hn bưc, ta s có:
1
1 2 2 3
1 2 1 2 1
0 , ,..., ...
n n n
l l l
l l l l
k n n n
f x x x



Vy, t các kết qu trên, ta có:
1 2 2 3 1 2 2 3 1 2 2 3
1 2 1 2 1 2 1 2
, ,..., ... ... ... ...
n n n
a b l
a a a a b b b b l l l l
n n n n
f x x x
 
.
Vậy đa thức
12
, ,...,
n
h x x x
cần tìm là đa thức:
1 2 2 3 1 2 2 3 1 2 2 3
1 2 1 2 1 2 1 2
, ,..., ... ... ... ...
n n n
a b l
a a a a b b b b l l l l
n n n n
h x x x x x x x x x x x x
Tính duy nht.
Gi s có một đa thức
1
' ,...,
n
h x x
sao cho
11
' ,..., ,...,
nn
h f x x

.
Thế thì:
11
' ,..., ,...,
nn
hh
.
Áp dng h qu ca B đề 3 ta có:
11
,..., ' ,...,
nn
h x x h x x
.
H qu. Gi s
1
1
...
nn
n
f x x a x a
là một đa thức bc
n
trên trưng
K
, có
n
nghim
12
, ,...,
n
trong trưng
E
nào đó cha
K
như một trưng con và gi s
12
, ,...,
n
g x x x
12
, ,...,
n
K x x x


là đa thc đi xng. Khi đó:
12
, ,...,
n
gK
.
Chng minh:
Tht vy! Theo định lý cơ bản v đa thức đi xng, tn ti
12
, ,...,
n
K x x x


sao cho
1 2 1 2
, ,..., , ,...,
nn
g x x x
.
Mt khác, theo công thc Viète, ta có:
October 4, 2021
[NGUYN THÀNH NHÂN TRƯỜNG ĐI HC AN GIANG]
19
Nghiên cu đnh lý Viète và ng dng | mathvn.com
12
, ,..., 1
k
k n k
aK
.
Bi vy
1 2 1 1 1 1 2
, ,..., ,..., ,..., ,..., , ,..., 1
n
n n n n n
g a a a K
.
Phép chng minh Định lý 2 cho phép chúng ta biết cách biu din mt đa thc đối xng qua
các đa thc đi xứng cơ bản. Trong thc tế để vic biu diễn nhanh chóng hơn, chúng ta có
nhn xét rằng đa thức đi xng
12
, ,...,
n
f x x x
có th không phi là đng cấp, nhưng các hạng
t có cùng mt cp ca nó lp thành một đa thức đi xng đẳng cấp, do đó
12
, ,...,
n
f x x x
tng ca những đa thức đối xứng đẳng cp.
Bây gi gi s
1 2 1 2
, ,..., , ,...,
nn
f x x x A x x x


là đa thc đi xng đẳng cp bc
k
và hng
t cao nht là
12
12
...
n
a
aa
n
x x x
.
Bc ca
12
, ,...,
n
f x x x
12
...
n
a a a k
.
Các đa thức đi xứng cơ bản
1
,...,
n

có bc theo th t
1,2,...,n
, nên đa thức tích
1 2 2 3
12
...
n
a
a a a a
n
a

cũng là đng cp và có bc là:
1 2 2 3 1 2
2 ... ...
nn
a a a a na a a a k
.
Do đó theo Định lý 2, ta có:
1 2 2 3
1 1 2 1 2 1 2
, ,..., , ,..., ...
n
a
a a a a
n n n
f x x x f x x x



cũng là đng cp bc
k
nếu khác
0
.
Sp xếp
1 1 2
, ,...,
n
f x x x
theo li t điển và gi s hng t cao nht là
October 4, 2021
[NGUYN THÀNH NHÂN TRƯỜNG ĐI HC AN GIANG]
20
Nghiên cu đnh lý Viète và ng dng | mathvn.com
12
12
...
n
b
bb
n
x x x
Thế thì
1 2 1 2
... ...
nn
a a a b b b k
1 2 1 2
, ,..., , ,...,
nn
a a a b b b
.
Theo Định lý 2, ta có dãy hu hn
1 2 1 2 1 2
, ,..., , ,..., , ,..., ... 8
n n n
a a a b b b c c c
trong đó
12
12
12
...
...
...
.........................
n
n
n
a a a
b b b
c c c
1 2 1 2 1 2
... ... ... ...
n n n
a a a b b b c c c k
.
Tp hp các phn t ca dãy
*
là mt b phn ca tp hp hu hn
11 1 1
,..., ,..., ,...,
n m mn
M t t t t
Trong đó
12
...
i i in
t t t
1 2 1 2
... ...
i i in n
t t t a a a k
.
Vy, theo Định lý 2 thì
1 2 2 3
1 2 1 2
1
, ,..., ...
i i i i in
m
t t t t t
n i n
i
f x x x

.
các h s
i
A
tìm được nh phương pháp hệ s bt đnh.
Chú ý: Nếu phn t
1
,...,
i in
tt
không có mt trong y
8
thì
0
i
.
Tp hp
11 1 1
,..., ,..., ,...,
nm
M t t t tn
gi là h thng s ca đa thc
12
, ,...,
n
f x x x
.
October 4, 2021
[NGUYN THÀNH NHÂN TRƯỜNG ĐI HC AN GIANG]
21
Nghiên cu đnh lý Viète và ng dng | mathvn.com
D. MT S NG DNG CỦA ĐỊNH LÝ VIÈTE.
I. Mt s ng dng.
Dng 1. Tìm hai s khi biết tng và tích.
Dng 2. Tính giá tr biu thc đi xng.
Dạng 3. Tìm điều kin ca tham s để hai nghim liên h vi nhau bi mt h thc
cho trước.
Dng 4. Tìm h thc liên h gia các nghim độc lp vi tham s.
Dng 5. Thiết lập phương trình bậc hai.
Dng 6. Xét du các nghim.
Dng 7. Gii h phương trình đi xng loi 1.
Dng 8. Chng minh bt đng thc.
Dng 9. ng dng trong bài toán cc tr.
Dng 10. ng dng trong bài toán tiếp tuyến.
Dng 11. ng dng h thc truy hi.
Dng 12. ng dng tính các biu thc lưng giác.
Dng 13. So sánh nghim.
Dng 14. ng dng khác.
II. Bài tp áp dng.
Dng 1. Tìm hai s khi biết tng và tích.
Câu 1. Tìm hai s
a
b
khi biết tng
S
và tích
P
:
22
2Sx
P x y

.
Gii
Ta có:
,X a X b
là nghim của phương trình:
2 2 2
20X xX x y
.
October 4, 2021
[NGUYN THÀNH NHÂN TRƯỜNG ĐI HC AN GIANG]
22
Nghiên cu đnh lý Viète và ng dng | mathvn.com
Ta có:
2 2 2 2
'0x x y y
.
Khi đó:
2
X x y x y
.
Vy:
; ; , ;a b x y x y x y x y
.
Câu 2. Tìm hai s
a
b
biết:
a)
22
9
41
ab
ab


.
Gii
Ta có:
22
2
22
81
81 41
9 81 2 81 20
22
ab
a b a b a b ab ab

.
Khi đó, ta có:
9
20
ab
ab

.
Như vậy,
,x a x b
là hai nghim thc của phương trình:
2
9 20 0xx
.
Ta có:
2
4
9 20 0
5
x
xx
x
.
Suy ra:
, 4;5 , 5;4ab
.
b)
5
36
ab
ab

.
Ta có:
2 2 2
25 4 25 25 4 25 4.36 169a b a b ab a b ab
.
Như vậy:
13
36
ab
ab
.
Suy ra:
,x a x b
là hai nghim mỗi phương trình
2
13 36 0xx
hoc
2
13 36 0xx
.
October 4, 2021
[NGUYN THÀNH NHÂN TRƯỜNG ĐI HC AN GIANG]
23
Nghiên cu đnh lý Viète và ng dng | mathvn.com
+) Phương trình
2
13 36 0xx
có hai nghim là:
4
9
x
x
.
Do vy:
, 4;9 , 9;4ab
.
+) Phương trình
2
13 36 0xx
có hai nghim là:
4
9
x
x


.
Do vy:
, 9; 4 , 4; 9ab
.
Dng 2. Tính giá tr biu thc đi xng.
Câu 1. Gi s
1 2 3
,,x x x
lần lượt là ba nghim của phương trình
3
0x px q
.
Tính giá tr ca biu thc:
2 2 2
1 2 2 3 1 3
S x x x x x x
.
Gii
Đặt:
2 2 2
1 2 3 1 2 1 3 2 3
,,g x x x x x x x x x
D thy:
2 2 2
1 2 3 1 2 1 3 2 3
,,g x x x x x x x x x
là đa thc đi xng.
H thng s mũ:
4;2;0 ; 4;1;1 ; 3;3;0 ; 3;2;1 ; 2;2;2M
.
Khi đó:
2 2 3 3 2
1 2 3 1 2 1 3 2 1 2 3 3
,,g x x x a b c d
.
Chn:
+)
11
22
33
10
1 1 4
00
x
xb
x







.
+)
1
1
2
23
3
0
2
3 27
1
2
x
d
xx

.
October 4, 2021
[NGUYN THÀNH NHÂN TRƯỜNG ĐI HC AN GIANG]
24
Nghiên cu đnh lý Viète và ng dng | mathvn.com
+)
1
1
2
23
3
3
1
04
2
4
x
a
xx


.
+)
1
1
2
23
3
1
1
1 18
1
1
x
c
xx


.
Khi đó:
2 2 3 3 2
1 2 3 1 2 1 3 2 1 2 3 3
, , 4 4 18 27g x x x
.
Vì:
1 2 3
,,x x x
lần lượt là ba nghim của phương trình
3
0x px q
nên
1
2
3
0
p
q

.
Do đó,
32
4 27S p q
.
Câu 2. Gi s
,,x y z
là ba nghim của phương trình
3
2021 2022 0

. Tính giá tr
ca biu thc
4 4 4
S x y z
?
Gii
Theo đnh lý Viète, ta có:
0
2021
2022
x y z
xy yz xz
xyz

.
Mt khác, ta có:
2
2 2 2
2
2
2 2 2
4 4 4 2 2 2 2 2 2 2 2 2 2 2 2 2 2 2
4 4 4 2 2 2 2 2 2 2 2 2 2 2 2
4 4 4
0 0 2
2
2 2 2 4 2 2 2
2 2 2 2 4
2
x y z x y z x y z xy yz xz
x y z xy yz xz
x y z x y y z x z x y y z x z xy z xyz x yz
x y z x y y z x z xy z xyz x yz x yz xy z xyz
x y z xy yz x


22
2
4 2 2.2021z xyz x y z xy yz xz
October 4, 2021
[NGUYN THÀNH NHÂN TRƯỜNG ĐI HC AN GIANG]
25
Nghiên cu đnh lý Viète và ng dng | mathvn.com
Dạng 3. Tìm điều kin ca tham s để hai nghim liên h vi nhau bi mt h thc
cho trước.
Câu 1. Cho phương trình
2
0y my p
có hai nghim là
1
y
2
y
. Định
m
p
để
1
1
1 y
2
1
1 y
cũng là nghim ca phương trình này.
Gii
Xét phương trình
2
0*y my p
.
Phương trình
*
có nghim khi và ch khi:
2
40mp
hay
2
4mp
.
Áp dng h thc Viète, ta có:
12
12
y y m
y y p
.
Khi đó, ta có:
12
12
1 2 1 2
12
2
1 1 2
1 1 1
1
1 1 1
.
1 1 1
yy
m
y y p m
y y y y
y y p m

.
Do
12
11
;
11yy
cũng là nghiệm của phương trình
*
nên:
2
1
1
1
2
1
m
m
pm
p
pm



.
T
1 , 2
, suy ra:
23p m m
.
+) Nếu
2 .0 2mp
(vô lý).
+) Nếu
2
2
m
mp
m
, thay vào
2
, ta được:
2
2
1 . 1 4 2 2
22
mm
m m m m m
mm




.
October 4, 2021
[NGUYN THÀNH NHÂN TRƯỜNG ĐI HC AN GIANG]
26
Nghiên cu đnh lý Viète và ng dng | mathvn.com
2
1
1 2 4 0
15
m
m m m
m
.
Th li:
+) Vi
1m
, thì
1p 
tha điu kin
2
4mp
.
+) Vi
15m 
thì
35
2
p
tha điu kin
2
4mp
.
+) Vi
15m 
thì
35
2
p
tha điu kin
2
4mp
.
Vy các cp
,mp
cn tìm là
3 5 3 5
1; 1 , 1 5; , 1 5,
22

.
Câu 2. Cho phương trình
22
2 6 0x mx m m
(
m
là tham s).
1. Vi giá tr nào ca
m
thì phương trình đã cho có hai nghiệm
1
x
2
x
sao cho
12
21
18
7
xx
xx

.
2. Vi giá tr nào ca
m
để phương trình đã cho có hai nghim
1
x
2
x
sao cho
12
8xx
.
Gii
1. Để phương trình
22
2 6 0x mx m m
có hai nghim thì:
22
' 6 6 0 6 1m m m m m
.
Vi điu kin
1
, ta có:
2
22
1 2 1 2
1 2 1 2
12
2 1 1 2 1 2
2
18 18 18
,0
7 7 7
x x x x
x x x x
xx
x x x x x x

.
October 4, 2021
[NGUYN THÀNH NHÂN TRƯỜNG ĐI HC AN GIANG]
27
Nghiên cu đnh lý Viète và ng dng | mathvn.com
22
2
22
4 2 6
18 6 9
, 2, 3
77
66
m m m
mm
mm
m m m m

.
1
2
2
4
8 48 0
12
m
mm
m

(tha điu kin
1
và đều khác
2
3
).
2. Vi điu kin
1
,
2
22
1 2 1 2 1 2 1 2 1 2 1 2
8 2 64 2 2 64 2x x x x x x x x x x x x
.
Nếu
1
x
2
x
cùng du thì:
2
12
6
62
03
6 2 3 0
3
m
m
xx
m m m m
m

.
Khi đó:
2
2
12
2 64 4 64 4x x m m
(tha điu kin
3
).
Nếu
1
x
2
x
trái du thì:
2
12
0 6 2 3 0 2 3 4x x m m m m m
.
Khi đó:
2
22
1 2 1 2
2 4 64 4 4 6 64 10x x x x m m m m
(không tha điu kin
4
).
Vậy để
12
8xx
thì:
4m 
.
Dng 4. Tìm h thc liên h gia các nghim độc lp vi tham s.
Câu 1. Gi s phương trình
2
2 3 4 0mx m x m
(
m
là tham s) có hai nghim
thc phân bit là
12
,xx
. Tìm h thc liên h gia
12
,xx
không ph thuc vào
m
.
Gii
Theo đnh lý Viète, ta có:
1 2 1 2
1 2 1 2
2 3 3 12
2 4 8
4 4 12
1 3 3
m
x x x x
m m m
m
x x x x
m m m







.
October 4, 2021
[NGUYN THÀNH NHÂN TRƯỜNG ĐI HC AN GIANG]
28
Nghiên cu đnh lý Viète và ng dng | mathvn.com
Suy ra:
1 2 1 2 1 2 1 2
4 3 11 4 4 3 11x x x x x x x x
là h thc liên h gia
12
,xx
,
độc lp vi tham s
m
.
Câu 2. Cho phương trình
2
1 2 1 0m x m x m
.
a) Gii và bin luận phương trình.
b) Khi phương trình có hai nghiệm phân bit
12
,xx
. Tìm mt h thc liên h gia
12
,xx
độc lp vi
m
.
Gii
a) Xét
1m
, phương trình đã cho tr thành:
1
4 1 0
4
xx
là nghim duy nht.
Vi
1m
, ta có:
2
' 1 1 3 1m m m m
.
+) Vi:
1
3
m 
, phương trình đã cho có hai nghiệm phân bit.
+) Vi
1
3
m 
, phương trình đã cho có một nghim kép.
+) Vi
1
3
m 
, phương trình vô nghim.
b) Theo đnh lý Viète, ta có:
12
12
21
4
2
11
1
1
11
m
xx
mm
m
xx
mm


lấy phương trình trên tr đi
4
lần phương trình dưới, ta có:
1 2 1 2
42x x x x
.
Vy h thc liên h gia
12
,xx
, độc lp vi tham s
m
là:
1 2 1 2
42x x x x
.
Dng 5. Thiết lập phương trình bậc hai.
Câu 1. Tìm phương trình bậc hai có hai nghim
2021x
2022x
.
Gii
October 4, 2021
[NGUYN THÀNH NHÂN TRƯỜNG ĐI HC AN GIANG]
29
Nghiên cu đnh lý Viète và ng dng | mathvn.com
Ta có:
12
12
4043
4086462
S x x
P x x

.
Như vậy, d dàng suy ra được, phương trình bc hai có hai nghim
2021x
2022x
là:
2
4043 4086462 0xx
.
Câu 2. Gi s
12
,xx
là hai nghim của phương trình
2
0x px q
. Hãy lp một phương
trình bc hai có hai nghim là
12
xx
12
xx
.
Gii
Theo đnh lý Viète, ta có:
12
12
x x p
x x q
.
Bài toán đã cho được quy v việc tìm phương trình bậc hai nhn
1 1 2
2 1 2
X x x
X x x

làm nghim.
Đặt:
12
12
S X X p q
P X X pq
.
Như vậy! Phương trình bậc hai cn tìm là:
2
0x p q x pq
.
Dng 6. Xét du các nghim.
Câu 1. Cho hàm s
2
33
:
2
xx
Cy
x

.
Tìm các giá tr ca
m
để đường thng
:d y mx m
ct đ th ti hai đim thuc v hai
nhánh ca đ th
C
.
Gii
Phương trình hoành đ giao điểm ca hai đ th
d
C
:
2
33
*
2
xx
mx m
x


.
2
* 1 3 2 3 0 1m x m x m
.
October 4, 2021
[NGUYN THÀNH NHÂN TRƯỜNG ĐI HC AN GIANG]
30
Nghiên cu đnh lý Viète và ng dng | mathvn.com
Để
d
ct
C
ti hai đim thuc hai nhánh khi và ch khi
1
có hai nghim
12
,xx
tha
mãn
12
2xx
hay
12
2 0 2xx
.
Đặt:
2tx
, ta đưa
1
v phương trình ẩn
t
:
2
1 3 1 1 0 2m t m t
.
Phương trình
2
phi có hai nghim trái du.
Khi đó:
1 . 1 0 1mm
.
Câu 2. Cho phương trình
32
1 3 1 4 1 0 1m x m x x m
(
m
là tham s).
Vi giá tr nào ca
m
, thì phương trình
1
có ba nghim phân biệt, trong đó có hai
nghim âm.
Gii
Ta có:
32
1 3 1 4 1 0 1m x m x x m
.
3 2 2
22
2
2
2
1 1 4 4 1 0
1 1 4 1 1 0
1 1 4 1 1 1 0
1
1 1 4 4 1 0
1 4 4 1 0
m x m x mx m x
m x x m x x
m x x m x x x
xa
x m x mx m
g x m x mx m b


Để phương trình
1
có ba nghim thc phân biệt thì phương trình
b
phi có hai
nghim thc phân bit khác
1
, tương đương với:
1
' 1 3 0
10
b
m
m
g

.
October 4, 2021
[NGUYN THÀNH NHÂN TRƯỜNG ĐI HC AN GIANG]
31
Nghiên cu đnh lý Viète và ng dng | mathvn.com
Hay:
1
11
1, 0, *
33
90
m
m m m m
m

.
Vi điu kin
*
, phương trình
1
có ba nghim phân biệt, trong đó có một nghim
10x 
và hai nghim còn li
1 2 1 2
,x x x x
là nghim ca
b
. Do đó để
1
có ba
nghim phân biệt trong đó có hai nghim âm thì:
12
0xx
, tương đương với:
12
12
1
41
1
1
0
1
**
4
1
4
0
1
4
1
0
m
m
m
P x x
m
m
m
m
S x x
m
m
m








.
Để phương trình
1
có ba nghim phân biệt, trong đó có hai nghiệm âm thì điều kin cn
và đủ là:
1m 
hoc
11
43
m
.
Dng 7. Gii h phương trình đi xng.
Câu 1. Gii h phương trình:
30
35
x y y x
x x y y


.
Gii
Đặt:
0
0
ux
vy


, h đã cho trở thành:
22
3
33
30
30
35
3 35
uv u v
u v uv
uv
u v uv u v





.
October 4, 2021
[NGUYN THÀNH NHÂN TRƯỜNG ĐI HC AN GIANG]
32
Nghiên cu đnh lý Viète và ng dng | mathvn.com
Tiếp theo ta đặt:
2
,4
S u v
SP
P uv

.
Ta thu đưc mt h mi:
33
30 30
5
6
3 35 125
SP SP
S
P
S SP S






(tha mãn).
Theo đnh lý Viète, ta có:
,uv
là nghim của phương trình:
2
5 6 0tt
.
Khi đó:
2
3
2
3
3
2
u
v
t
t
u
v
.
Dẫn đến nghim ca h
; 4;9 ; 9;4xy
Câu 2. Gii h phương trình:
22
44
32
5
x xy y
xy

.
Gii
Ta có:
2
4 4 2 2 2 2
2x y x y x y
.
Đặt:
22
u x y
v xy

, h đã cho trở thành:
22
32
25
uv
uv

.
Gii h đã cho, ta được:
3
2
9 4 2
6 3 2
u
v
II
u
v


.
Vi h
I
, thì:
22
22
22
3
3
2
2
xy
xy
xy
xy





.
October 4, 2021
[NGUYN THÀNH NHÂN TRƯỜNG ĐI HC AN GIANG]
33
Nghiên cu đnh lý Viète và ng dng | mathvn.com
Theo đnh lý Viète, thì:
22
,xy
là nghim của phương trình:
2
3 2 0tt
, ta được:
1
2
t
t
.
Thế vào hệ, ta được:
2
2
2
2
1
2
0
2
1
0
x
y
xy
x
y
xy
.
Suy ra nghim
; 1; 2 ; 1; 2 ; 2; 1 ; 2;1xy
.
Trưng hp còn li vô nghim.
Dng 8. Chng minh bt đng thc.
Câu 1. Cho
,,x y z
khác
0
, tha mãn:
x y z xyz
2
x yz
.
Chng minh rng:
2
3x
.
Gii
T gi thuyết, ta có:
3
2
y z x x
yz x
.
Theo đnh lý Viète, thì
,yz
là nghim của phương trình:
2 3 2
0t x x t x
.
Do tn ti các s
,yz
, nên phương trình trên phi có nghim:
Tc là:
22
3 2 2 2
0 4 0 1 4 0x x x x x



.
Vì:
0x
, nên:
2
2
22
2
12
1 4 0 3
12
x
xx
x




.
October 4, 2021
[NGUYN THÀNH NHÂN TRƯỜNG ĐI HC AN GIANG]
34
Nghiên cu đnh lý Viète và ng dng | mathvn.com
Câu 2. Cho các s thc
,,x y z
tha mãn:
5
8
x y z
xy yz xz
. Chng minh rng:
7
1 , ,
3
x y z
.
Gii
T gi thuyết, ta xem
z
là tham s, ta có h phương trình ẩn
,xy
:
55
8 8 5
x y z x y z
xy z x y xy z z





.
Theo đnh lý Viète thì
,xy
là nghim của phương trình:
2
5 8 5 0t z t z z
.
Do phương trình có nghiệm đối vi
,xy
nên:
2
7
5 4 8 5 0 1
3
z z z z


.
Do vai trò bình đng ca
,,x y z
nên ta có kết lun tương tự đối vi
x
y
.
Dng 9. ng dng trong bài toán cc tr.
Câu 1. Tìm tt c các giá tr tham s
m
để đồ thm s
32
2 1 3 y f x mx m x m x
có hai đim cc tr có hoành đ dương là?
Gii
Ta có:
2
3 2 2 1f x mx m x m
.
Để đồ th hàm s có hai điểm cc tr có hoành độ dương thì:
2
1
2 3 1 0
4
0
2
22
1
0 0 2 0 0
32
0
0
1
0
1
m m m
m
m
S m m
m
P
m
m
m
m




.
Câu 2. Tìm tt c các giá tr nguyên ca tham s
10;10m
để hàm s:
October 4, 2021
[NGUYN THÀNH NHÂN TRƯỜNG ĐI HC AN GIANG]
35
Nghiên cu đnh lý Viète và ng dng | mathvn.com
2
32
3
4 1 1
2
y x mx m x
có hai đim cc tr
12
,xx
tha mãn
1 2 1 2
x x x x
?
Gii
Ta có:
2
2
9
81
2
y x mx m
.
Hàm s
y
có hai đim cc tr khi phương trình
0y
có hai nhgim phân bit
23 18 9mm
.
Theo đnh lý Viéte ta có:
12
16
9
m
xx
;
2
12
21
9
m
xx
.
1 2 1 2
x x x x
2
2
5 2 6
2 16
1 10 1 0
99
5 2 6
m
m m m m
m


.
10;0 10m
.
Dng 10. ng dng trong bài toán tiếp tuyến.
Câu 1. Cho hàm s
2
1
x
y
x
có đ th
C
và điểm
0;Aa
. Hi có tt c bao nhiêu giá
tr nguyên ca
a
để t điểm
A
k được hai tiếp tuyến đến
C
sao cho hai tiếp điểm nm
v hai phía ca trc hoành?
Gii
Ta có:
2
3
'
1
y
x
.
Gi tiếp điểm là
0
0
0
2
;
1
x
Mx
x



. Khi đó phương trình tiếp tuyến ca
C
ti
M
là:
0
0 0 0 0
2
0
0
2
3
:
1
1
x
d y f x x x y x x
x
x
.
October 4, 2021
[NGUYN THÀNH NHÂN TRƯỜNG ĐI HC AN GIANG]
36
Nghiên cu đnh lý Viète và ng dng | mathvn.com
Vì đưng thng
0
0
2
0
0
2
3
:
1
1
x
d y x x
x
x
đi qua điểm
0;Aa
. Khi đó:
22
00
0 0 0 0 0
2
0
0
2
0 0 0
32
3 2 2
1
1
1 2 2 2 0, 1 1
xx
a x x x ax ax a
x
x
a x a x a x
T
A
k được 2 tiếp tuyến đến
C
Phương trình
1
có 2 nghim
0
x
phân bit khác
1
2
2 1 2 0
3 6 0
2
30
1 .1 2 2 .1 2 0
a a a
a
a
a a a


.
Khi đó phương trình (1) có hai nghim
12
,1xx
.
Hai tiếp điểm nm v hai phía ca trc hoành
1 2 1 2 1 2
12
1 2 1 2 1 2
22
2
24
2 2 2 4
11
. 0 0 0 0
22
1 1 1
2
1
11
a
a
x x x x x x
aa
yy
a
x x x x x x
a
aa




2 4 8 4 4
9 6 2
1
0 0 3 2 0
2 2 4 1
33
1
a a a
a
a
aa
a a a
a
.
Câu 2. Cho hàm s:
1
:
21
x
Cy
x

. Đường thng
:d y x m
. Vi mi
m
ta luôn có
d
ct
C
tại hai điểm phân bit
,AB
. Gi
12
,kk
lần lượt là h s góc ca tiếp tuyến vi
C
ti
,AB
. Tìm
m
để tng
12
kk
đạt giá tr ln nht.
Gii
Phương trình hoành đ giao điểm ca
d
C
là:
2
1
1
2
21
2 2 1 0 *
x
x
xm
x
g x x mx m

.
Theo đnh lý Viète, ta có:
12
12
1
2
x x m
m
xx

.
October 4, 2021
[NGUYN THÀNH NHÂN TRƯỜNG ĐI HC AN GIANG]
37
Nghiên cu đnh lý Viète và ng dng | mathvn.com
Gi s:
1 1 2 2
; , ;A x y B x y
.
Ta có:
2
1
'
21
y
x
.
Nên tiếp tuyến ca
C
ti
A
B
có h s góc lần lưt là:
1
2
1
2
2
2
1
21
1
21
k
x
k
x


.
Vy:
22
1 2 1 2
12
2 2 2
12
1 2 1 2
4 4 2
11
2 1 2 1
4 2 1
x x x x
kk
xx
x x x x



.
2
2
1 2 1 2 1 2
12
22
1 2 1 2
2
22
12
4 2 4 2
4 1 4 2
2 2 2 1
4 2 1
4 8 6 4 2 1 2 4 1 2 2
x x x x x x
m m m
kk
mm
x x x x
k k m m m m m





Vy tng
12
kk
đạt giá tr ln nht bng
2
khi
1m 
.
Dng 11. ng dng h thc truy hi.
Phương pháp truy hi:
Xét phương trình bậc hai:
22
0, 0ax bx c a
.
Gi s
12
,xx
là các nghim ca nó.
Đặt:
12
,
nn
n
S x x n
.
Lúc đó ta có h thc truy hi tuyến tính sau:
21
01
n n n
aS bS cS

.
October 4, 2021
[NGUYN THÀNH NHÂN TRƯỜNG ĐI HC AN GIANG]
38
Nghiên cu đnh lý Viète và ng dng | mathvn.com
Chng minh:
Ta có:
2 2 1 1
2 1 2 1 2 1 2 1 2 1 2
n n n n n n
n
S x x x x x x x x x x
.
2 1 2 1
. . 0
n n n n n n
bc
S S S aS bS cS
aa



(đpcm).
Câu 1. Tìm s nguyên ln nhất không vưt quá
7
4 15
Gii
Ta đt:
1
2
4 15
4 15
x
x


.
Khi đó:
12
,xx
là nghim của phương trình:
2
8 1 0xx
.
Đặt:
12
,
nn
n
S x x n
. Ta có h thc:
21
80
n n n
S S S

.
Ta tính đưc:
1
2
3
4
5
6
7
8
62
488
3842
30248
238142
1874888
S
S
S
S
S
S
S
.
Như vậy:
77
12
1874888xx
. Mà
7
2
01x
.
Suy ra:
7
1
1874887 18748888x
.
Vy s nguyên ln nht không vưt quá
7
4 15
1874887
.
Câu 2. Tìm ch s tn cùng ca phn nguyên ca s
2021
5 3 3
.
October 4, 2021
[NGUYN THÀNH NHÂN TRƯỜNG ĐI HC AN GIANG]
39
Nghiên cu đnh lý Viète và ng dng | mathvn.com
Gii
Ta đt:
1
12
2
5 3 3
,
5 3 3
x
xx
x


là nghim của phương trình:
2
10 2 0xx
.
Đặt:
12
,
nn
n
S x x n
. Theo h thc truy hi, ta có:
21
10 2
n n n
S S S


.
Ta có:
1 2 1
10 10
k
SS

.
Để ý rng:
2
10x
, nên suy ra:
12
1
nn
n n n
S x S x S
.
Vy nên
1
n
n
xS


. Vì
2021
là s l nên
2021
10 S
.
Vy ch s tn cùng ca phn nguyên ca s
2021
5 3 3
là s
0
.
Dng 12. ng dng tính các biu thc ng giác.
Câu 1. Chng minh rng:
9 9 17 17 3
cos cos cos cos cos cos
12 12 12 12 12 12 4
.
Gii
Áp dng công thc nhân ba, ta có:
3
cos3 4cos 3cosx x x
.
+) Vi
12
x
, ta có:
3
2
4cos 3cos 0 1
12 12 2

.
Nên:
cos
12
là nghim của phương trình:
3
2
4 3 0
2
tt
.
Lp luận tương tự, ta có:
9
cos
12
17
cos
12
cũng là nghim của phương trình
1
.
Theo đnh lý Viète, ta có:
9 9 17 17 3
cos cos cos cos cos cos
12 12 12 12 12 12 4
.
October 4, 2021
[NGUYN THÀNH NHÂN TRƯỜNG ĐI HC AN GIANG]
40
Nghiên cu đnh lý Viète và ng dng | mathvn.com
Câu 2. Cho
0b
, gi s phương trình:
32
0x ax x b
có ba nghim là
1 2 3
,,x x x
. Chng minh rng:
1 2 2 3 3 1
1 2 2 3 3 1
1 1 1 1 1 1
4x x x x x x
x x x x x x
.
Gii
Theo đnh lý Viète, ta có:
1 2 2 3 3 1
1x x x x x x
.
Đặt:
1 2 3
tan ; tan ; tanx x x
, thế thì ta có:
tan tan tan tan tan tan 1
.
Suy ra:
,
2
kk
, suy ra:
2 2 2 ,kk
.
Đẳng thc cn chứng minh tương đương vi:
tan cot tan cot tan cot tan cot tan cot tan cot 4
Để ý rng:
tan cot 2cot2x x x
.
thế, đẳng thc cn chng minh tr thành:
cot 2 cot 2 cot2 cot2 cot 2 cot 2 1
.
Đẳng thc hiển nhiên đúng, vì:
2 2 2 ,kk
(đpcm).
Vy:
1 2 2 3 3 1
1 2 2 3 3 1
1 1 1 1 1 1
4x x x x x x
x x x x x x
.
October 4, 2021
[NGUYN THÀNH NHÂN TRƯỜNG ĐI HC AN GIANG]
41
Nghiên cu đnh lý Viète và ng dng | mathvn.com
Dng 13. So sánh nghim.
Định lý v du kết hợp định lý Viéte cho tam thc bc hai:
Đặt: .
So sánh nghim vi hng s .
Điu kiện để: là: .
Điu kiện để: là: .
Điu kiện để: là: .
So sánh nghim vi hng s (Vi là các tham s thực cho trước).
Điu kiện để: là: .
Điu kiện để: là: .
Điu kiện để: là: .
12
12
.
S x x
P x x

.a
0
i
12
0xx
1
0 ' 0
0
0
S
P
ii
12
0 xx
2
0 ' 0
0
0
S
P
iii
12
0xx
0 3P
.b
,

,

i
12
xx

.04af
ii
12
xx

5
0 ' 0
.0
2
af
S
iii
12
xx

6
0 ' 0
.0
2
af
S
October 4, 2021
[NGUYN THÀNH NHÂN TRƯỜNG ĐI HC AN GIANG]
42
Nghiên cu đnh lý Viète và ng dng | mathvn.com
Điu kiện để: là: .
Điu kiện để: là: .
Điu kiện để: là: .
Điu kiện để: là: .
Để tiết kim thi gian cho mt s bài toán ta có th tính trc tiếp da trên hàm s bc ba:
.
Khi đó:
Hàm s đồng biến trên khi và chi khi .
m s nghch biến trên khi và chi khi .
iv
12
xx

7
.0
.0
af
af
v
12
xx

8
.0
.0
af
af
vi
12
12
xx
xx


0 9.ff

vii
12
xx

0
0 ' 0
.0
.0
2
1
af
af
S


32
0y ax bx cx d a
'2
'
0
30
f
a
b ac
'2
'
0
30
f
a
b ac
October 4, 2021
[NGUYN THÀNH NHÂN TRƯỜNG ĐI HC AN GIANG]
43
Nghiên cu đnh lý Viète và ng dng | mathvn.com
Extra Techniques
Study tips
Cơ sở hình thành các điu kin:
:Phương trình hai nghiệm phân bit nên hoc . Do tng ca hai s âm
là mt s âm và tích ca hai s âm là mt s dương.
:Phương trình hai nghim phân bit nên hoc . Do tng ca hai s
dương là một s dương và tích của hai s ơng là một s dương.
:S chỉ cn một điều kin khi hay
. Khi đó: . Đã thỏa mãn điều kin cần đủ để
phương trình có hai nghim thc phân bit.
: trong khong hai nghim thì trái du vi h s ngoài hai
khong nghim thì cùng du vi h s . Và tích ca mt s dương với mt s âm mt
s âm.
: nên : nên
: nên . Khi đó:
.
Lưu ý: các điều kin thì du ca các tích đều được xét
dấu tương tự vi . Và vì phương trình có hai nghiệm phân bit nên
hoc .
Câu 1. Tìm để hàm s đồng biến trên khong .
Gii
1
0
'0
2
0
'0
3
0P
00
c
PP
a
00ac ac
2
40b ac
4 , 7 , 8 , 9
gx
a
a
5
21
xx

1 2 1
22
2
S
x x x

6
12
xx

1 2 2
22
2
S
x x x

6
12
xx

1 1 2 2
22x x x x
22S


2
S

5 , 6 ,
10
.,af
.af
4,
7 , 8 , 9
0
'0
m
32
2
1 2 5
3
y x m x mx
0;2
October 4, 2021
[NGUYN THÀNH NHÂN TRƯỜNG ĐI HC AN GIANG]
44
Nghiên cu đnh lý Viète và ng dng | mathvn.com
Ta có: .
Để hàm s đồng biến trên khong thì phương trình có hai nghim thc phân
bit tha mãn: . Điều đó tương đương với h:
.
Câu 2. Tìm tt c các giá tr thc ca tham s để hàm s:
đồng biến trong khong ?
Gii
Ta có: .
Để hàm s đồng biến trên thì phương trình: có hai nghim phân bit
tha mãn: . Khi đó:
Dng 14. ng dng khác.
Câu 1. Cho parabol
2
:P y x
đưng thng
d
đi qua điểm
0; 1I
h s
góc
k
. Gi
A
B
các giao điểm ca
P
d
. Gi s
, AB
lần t hoành
độ
. Giá tr nh nht ca biu thc:
33
12
xx
là?
Gii
2
' 2 2 1 2y x m x m
0;2
'0y
12
xx
12
02xx
. 0 0
2.2 0 0
0
2 12 0 6
. 2 0
af
mm
m
mm
af



m
32
1
1 3 10
3
y x m x m x
0;3
2
' 2 1 3y x m x m
0;3
'0y
12
,xx
12
03xx
2
2
' 1 3 0
' 4 0
1 . 0 0 0 0
1 . 3 0 3 0
12
3 0 3
7
7 12 0 12
7
mm
mm
ff
ff
mm
m m m
m
m





October 4, 2021
[NGUYN THÀNH NHÂN TRƯỜNG ĐI HC AN GIANG]
45
Nghiên cu đnh lý Viète và ng dng | mathvn.com
Cho parabol
2
yx
và đường thng
d
đi qua điểm
0; 1I
và có h sc là
k
.
Gi
A
B
là các giao đim ca
P
d
. Gi s
A
B
lần lượt có hoành độ
12
,xx
.
+ Đường thng
d
có phương trình:
1y kx
.
+ Phương trình tương giao
d
P
:
22
1 1 0 *x kx x kx
.
+
*
luôn có
2
nghim phân bit:
12
;xx
, vì:
2
4 0,kx
.
Theo đnh lý Viéte, ta có:
12
,x x k
12
1xx 
.
Ta có:
3 3 2
1 2 1 2 1 2 1 2
( ) ( )x x x x x x x x


=
2
1 2 1 2 1 2
. ( )x x x x x x
.
Ta có:
2
2
2
1 2 1 2 1 2
44x x x x x x k
.
33
12
xx
=
22
4. 1 4.1 2kk
,
k
. Đẳng thc xy ra khi
0 k
.
Câu 2. Tìm tt các các giá tr thc
m
để đường thng
y x m
cắt đồ th hàm s
32
1
2 3 2 3
3
y x m x m x m
tại ba điểm phân bit
0; , , A m B C
sao cho đường
thng
OA
là phân giác ca góc
BOC
.
Gii
Phương trình hoành độ giao điểm:
32
2
0
1
2 3 2 3
1
3
2 6 8 0 *
3
x
x m x m x m x m
x m x m
.
Để đường thẳng cắt đồ thị tại ba điểm phân biệt thì
*
phải có hai nghiệm phân biệt khác
0x
Hay:
October 4, 2021
[NGUYN THÀNH NHÂN TRƯỜNG ĐI HC AN GIANG]
46
Nghiên cu đnh lý Viète và ng dng | mathvn.com
2
2
18 8 3
44
3
4
12 0
2 6 8 0
3
18 8 3
1
3
4
3
6 8 0
3
4
3
m
mm
mm
m
m
m
m



.
Suy ra: Tọa độ các điểm
,BC
lần lượt là:
1 1 2 2
, , ,B x x m C x x m
.
Theo định lý Viéte, ta :
12
12
32
3 6 8
x x m
x x m

.
Để ý:
OA Oy
(Do
0;0
0;
O Oy
A m Oy
) có véctơ chỉ phương
0;1 .j
Vậy để đường thẳng
OA
là phân giác của góc
BOC
.
12
2 2 2 2
1 1 2 2
cos , cos , 2
( ) ( )
m x m x
j OB j OC
x m x x m x

.
22
22
1 2 2 2 1 1
2 2 2 2
22
1 2 2 2 1 1
2 m x x m x m x x m x
m x x m x m x x m x
2 2 2 2
2
22
22
1 1 2 12 2 1
.m x x mm x m x mxx x m x
2
2 2 2 2 2 2 2 2 2 2 2 2 2
2 1 1 2 2 1 2 1 2 1 1 2 1 2
2 2 2 2
1 2 1 2 1 2 1 2 1 2 1 2 1 2
12
1 2 1 2 1 2
1 2 1 2
2
( ) 2 . 2
2 0 2 0
20
( ) 2
0
0
3 ( 2) 6(6 8)
3 42 48
x m x x m x x m mx x x x x m mx x x x
m x x mx x x x m x x x x mx x x x
mx mx
m x x m x x x x
m x x x x
m
m
m m m
mm





0
7 33
7 33
m
m
m

.
Đối chiếu điều kiện
1
0;0AO
nên ta nhận
7 33m 
.
October 4, 2021
[NGUYN THÀNH NHÂN TRƯỜNG ĐI HC AN GIANG]
47
Nghiên cu đnh lý Viète và ng dng | mathvn.com
Câu 3. Biết đồ thm s
2
1 1 7y x x x m
ct trc hoành ti
4
điểm phân
bit có hoành đ
1 2 3 4
, , ,x x x x
. Tìm tt c các giá tr nguyên ca tham s
m
để
1 2 3 4
1 1 1 1
1
1 1 1 1x x x x
?
Gii
Xét phương trình hoành độ giao điểm:
2 4 2
1 1 7 0 8 7 0 1x x x m x x m
Đặt:
2
,0t x t
. Khi đó phương trình đã cho có dạng:
2
8 7 0 2t t m
Vì đ th ct trc hoành ti
4
điểm phân biệt nên phương trình
1
4
nghim thc
phân biệt hay phương trình
2
có hai nghim phân biệt dương. Điều đó tương đương
vi:
' 0 16 7 0
0 8 0 9 7
0 7 0
m
Sm
Pm





.
Khi đó phương trình
2
có hai nghiệm dương:
12
0 tt
tho mãn:
12
12
8
7
tt
t t m


.
Suy ra phương trình
1
có bn nghim lần lượt là:
1 2 2 1 3 1 4 2
, , ,x t x t x t x t
.
Như vậy:
1 2 3 4
2 1 1 2
1 1 1 1 1 1 1 1
11
1 1 1 1
1 1 1 1
x x x x
t t t t
.
12
2 1 1 2 1 2
2 2 1 1
42
1 1 1 1 2 2
1 1 1
1 1 1
1 1 1 1
4 2.8 12 12
1 1 0 0 12
1 8 7
tt
t t t t t t
t t t t
m
m
m m m


Kết hợp điều kiện ta đưc:
0 7 1;2;3;4;5;6mm
.
October 4, 2021
[NGUYN THÀNH NHÂN TRƯỜNG ĐI HC AN GIANG]
48
Nghiên cu đnh lý Viète và ng dng | mathvn.com
Câu 4. Chom s
32
3 2 9 31 27 7y x m x m x m
có đ th
C
. Biết
rng ng vi giá tr nguyên
1
mm
thì hàm s
C
ct trc hoành ti
3
đim lp tnh
mt cp s cng có các phn t đều nguyên dương và ứng vi giá tr nguyên
2
mm
thì
hàm s ct trc hoành ti
3
điểm lp thành mt cp s nhân có các phn t đều nguyên
dương. Tìm giá trị
12
,mm
?
Gii
T hàm s:
32
3 2 9 31 27 7y x m x m x m
.
Ta nhn thy
1x
luôn luôn là mt nghim của phương trình
0y
.
Bng phép chia Hoocner ta có:
2
1 4 2 7 27y x x m x m
.
Giao đim ca hàm s vi trc hoành là nghim ca:
2
1
4 2 7 27 0
x
x m x m
.
Để hàm s ct trc hoành ti
3
điểm phân bit thì:
2
2
' 2 7 27 0
22
5
1 4 2 .1 7 27 0
mm
m
mm

.
Gi s
3
giao điểm ca
y f x
vi trc hoành lần lượt là:
12
1, ,xx
.
Khi đó:
-
3
đim trên lp thành mt cp s cng khi và ch khi:
1 1 2
2
2
2 1 2
2
2 3 2 4
2 3 2 4
1 2 4
3
2 3 1 7 3 27
1 2 . 7 27
2 3 1 7 27
2
3
3
; 2;6
2
2 3 2 4
2
21
7 45
2 3 1 6
;;
7
2
48
4
dm
dm
x d x x m
m
dd
x d x x m
d d m
d
m
d m TM
dm
d
d d d
d m L
d











Hay:
1
6m
.
-
3
đim trên lp thành mt cp s nhân khi và ch khi:
October 4, 2021
[NGUYN THÀNH NHÂN TRƯỜNG ĐI HC AN GIANG]
49
Nghiên cu đnh lý Viète và ng dng | mathvn.com
2
2
1
12
2
3
2
3
12
2
24
24
24
4
. 7 27
7 27
7 27
2
d d m
xd
x x m
d d m
dd
x x m
d
xd
dm








.
Nên:
2
5m
.
Câu 5. Cho hàm s
3
2
1
6 2021
3
y x mx m x
. Tìm tt c các giá tr nguyên ca
m
thuc để đồ th hàm s
5
điểm cc tr.
Gii
Đồ th hàm s
3
2
1
6 2021
3
y x mx m x
5
đim cc tr khi và ch khi đ th
hàm s
32
1
6 2021
3
y x mx m x
có hai đim cc tr nm bên phi trc
Oy
hay
hàm s
32
1
6 2021
3
y x mx m x
có hai đim cc tr dương.
Ta có:
2
26y x mx m
.
Bài toán đã cho trở thành vic tìm
m
để phương trình
2
2 6 0x mx m
có hai
nghiệm dương phân bit. Khi đó:
2
3
0
2
60
0 2 0 0 3
6 0 6
0
m
m
mm
b
m m m
a
mm
c
a






.
2
2
32
4
2
24
2
; 2;5
77
13 0
22
3 113
4
dd
m
d d m
d
d m TM
d d d
dL




October 4, 2021
[NGUYN THÀNH NHÂN TRƯỜNG ĐI HC AN GIANG]
50
Nghiên cu đnh lý Viète và ng dng | mathvn.com
Câu 6. Cho hàm s
32
, , ,y ax bx cx d a b c d
có đ th là đường cong như
hình vẽ. Định du ca các h s
, , ,a b c d
.
Gii
Ta có:
lim 0
x
ya


.
Gi
12
,xx
là hoành đ hai đim cc tr ca hàm s suy ra
12
,xx
nghiệm phương trình
2
3 2 0y ax bx c
nên theo định lí Viéte, ta có:
+) Tng hai nghim:
12
2
0 0 0
3
bb
x x b
aa
.
+) Tích hai nghim:
12
. 0 0
3
c
x x c
a
.
Li có đ thm s ct trc tung ti điểm có tung độ dương nên
0d
.
Câu 7. Cho hàm s
2
2
4 12
62
xx
y
x x k


đồ th
C
. Tìm tp hp
S
cha tt c các giá
tr thc ca thám s
k
để đồ th
C
đúng hai tim cận đứng?
Gii
Điu kin:
2
04
6 2 0
x
x x k

.
Ta có:
2
12 4 0,x x x D
.
Nên đ
C
có hai tim cận đứng thì phương trình:
22
6 2 0 6 2 0 *x x k x x k
có hai nghim phân bit thuc
0;4
.
October 4, 2021
[NGUYN THÀNH NHÂN TRƯỜNG ĐI HC AN GIANG]
51
Nghiên cu đnh lý Viète và ng dng | mathvn.com
Để phương trình
*
có hai nghim phân bit thì:
9
9 2 0
2
kk
.
Gi hai nghim phân bit ca
*
12
xx
, ta có:
12
04xx
.
Theo định lý Viéte ta có:
12
12
12
12
12
12
0
20
0
6
6 0 0
4
4 4 0
2
2 24 16 0 2 8 0
4 4 0
6 8 0
xx
k
xx
xx
k
k
xx
x x k
kk
xx







Kết hp nghim ta có:
9
4;
2
S


.
Câu 8. Vi giá tr nào ca tham s
m
thì phương trình
1
4 2 2 0
xx
mm
có hai
nghim
12
,xx
tha mãn
12
3xx
?
Gii
Ta có:
2
1
4 2 2 0 2 2 .2 2 0 *
x x x x
m m m m
.
Xem phương trình
*
là phương trình bc hai theo n
20
x
t 
.
Khi đó:
2
* 2 2 0 **t mt m
.
Điu kiện để phương trình
**
có hai nghiệm dương phân biệt:
2
' 2 0
2 0 2
20
mm
S m m
Pm

.
Theo đnh lý Viéte, ta có:
12
. 8 2 8 4t t m m
.
Note:
Để phương trình
*
có hai nghim
12
,xx
tha mãn yêu cầu đề bài thì
**
phi có hai
nghiệm dương phân bit tha mãn
12
.8tt
do
1 2 1 2
3
12
3 2 2 8 2 .2 8
x x x x
xx
.
Câu 8. Xác định
m
để bất phương trình sau có nghim:
2
2
10
2 1 4 1 0
x
x m x m

.
Gii
Đặt:
2
2
1 0 1
2 1 4 1 0 2
x
x m x m

.
October 4, 2021
[NGUYN THÀNH NHÂN TRƯỜNG ĐI HC AN GIANG]
52
Nghiên cu đnh lý Viète và ng dng | mathvn.com
T
2
2
11
1
2
x
mm
.
+) Trường hp 1:
2
0
h phương trình đã cho có nghim đúng với mi
1;1x
2
2 0 0;2 *m m m
.
+) Trường hp 2:
2
0
0
2
m
m
h phương trình có nghim
2
có nghim
2
21
12
2
0
0
2
2
0
2
1 6 4 0
. 1 0
3
21
2
1
1
2
1
2
1
0
0
0
2
2
. 1 0
1 2 0
21
1
1
2
2
m
m
m
m
fm
m
af
m
S
m
xx
xx
m
m
m
m
af
fm
S
m






























2
0
3
2*
0
0
0
m
m
m
m
m

.
T
* **
2
3
m
.
Câu 9. Cho các s thc
, , a b c
(vi
0)a
sao cho phương trình
2
0ax bx c
có hai
nghim thuc đon
0;1
. Tìm giá tr ln nht ca biu thc:
( )(2 )
()
a b a b
P
a a b c


.
Gii
Gi
12
,xx
là nghim ca phương trình đã cho. Theo định lý Viéte, ta có:
12
12
b
xx
a
c
xx
a
.
October 4, 2021
[NGUYN THÀNH NHÂN TRƯỜNG ĐI HC AN GIANG]
53
Nghiên cu đnh lý Viète và ng dng | mathvn.com
Do
0a
, nên:
1 2 1 2 1 2 1 2 1 2
1 2 1 2 1 2 1 2
12
1 2 2 1 1
11
1
bb
x x x x x x x x x x
aa
P
bc
x x x x x x x x
aa





2
22
22
1 2 1 2 1 2 1 2
1 2 1 2 1 2
1 2 1 2
1 2 1 2 1 2 1 2 1 2 1 2
21
21
2
1 1 1
x x x x x x x x
x x x x x x
x x x x
P
x x x x x x x x x x x x
Gi s
12
xx
do
2
nghim thuc
0;1
nên
22
1 1 2 2
1x x x x
.
1 2 1 2
10x x x x
nên ta có:
22
1 2 1 2 1 2 1 2
1 2 1 2 1 2 1 2
1
13
11
x x x x x x x x
P
x x x x x x x x
.
Vy
3max P
.
Dấu đẳng thc xy ra khi và ch khi:
1
2
2
1 1 2
2
1
2
2
0
0
1
0
1
1
0
1
2
x
c
x
x x x
ba
x
x
b
ac
x




Câu 10. Cho hàm s
32
21
2 2022
2
m
y x x x



. Biết rng tn ti hai giá tr tham s
12
mm
;
tha mãn hàm s đạt cc tr ti hai đim
12
,xx
sao cho
12
2 3 3x x m
. Tìm
12
,mm
.
Gii
3 2 2
21
2 2022; ' 3 2 1 2
2
m
y x x x y x m x



Hàm s có hai điểm cc tr khi phương trình
'0y
có hai nghim phân bit
22
1
2 1 6 0 2 1 24,(*)
4
mm
October 4, 2021
[NGUYN THÀNH NHÂN TRƯỜNG ĐI HC AN GIANG]
54
Nghiên cu đnh lý Viète và ng dng | mathvn.com
Theo đnh lý Viète ta có:
12
12
21
3
2
3
m
xx
xx

.
Vì:
1
12
12
2
1
21
32
5
3
1
2 3 3
11
15
m
xm
xx
x x m
xm






.
Thế vào
12
2
.
3
xx
, ta được:
2
12
2 1 1 2 1 2
3 2 . 11 3 31 22
3 5 15 3 75 3
x x m m m m
.
1
22
2
1
5 73 31
6
3 31 22 50 3 31 72 0
1
5 73 31
6
m
m m m m
m

.
Câu 11. Xác đnh giá tr ca tham s
m
để hàm s
32
1 3 1 2 4y f x m x m x mx
đồng biến trên khoảng có độ dài bng
1
.
Gii
Ta xét các kh năng:
Trường hp 1:
1m 
, khi đó:
24y f x x
là một đường thng có h sc
20k
. Nên hàm s nghch biến trên .
Trường hp 2:
1m 
, khi đó:
Nếu
'0fx
vô nghim hoc có nghim kép thì hàm s luôn đng biến trên . (Loi)
Nếu
'0fx
có hai nghim thc
12
,xx
thì hàm s đồng biến trên hai khong
1
;x
2
;x 
. (Loi)
Như vậy Trường hp 2 không tn ti giá tr thc
m
nào để cho hàm s đồng biến trên
khoảng có độ dài bng
1
.
Trường hp 3:
1m 
, khi đó:
October 4, 2021
[NGUYN THÀNH NHÂN TRƯỜNG ĐI HC AN GIANG]
55
Nghiên cu đnh lý Viète và ng dng | mathvn.com
Nếu
'0fx
vô nghim hoc có nghim kép thì hàm s luôn nghch biến trên .
(Loi)
Nếu
'0fx
có hai nghim thc
12
,xx
thì hàm s đồng biến trên khong
12
;xx
.
Như vậy để hàm s đồng biến trên khoảng có độ ln bng
1
thì:
'
12
1
' 0 *
1
fx
m
xx



.
Ta có:
2
' 3 1 6 1 2f x m x m x m
.
Suy ra:
2
'
1
' 9 1 6 1 1 3 9 0
3
fx
m
m m m m m
m


.
Theo đnh lý Viète, ta có:
12
12
2
2
31
xx
m
xx
m

.
22
1 2 1 2 1 2 1 2
1 1 4 1x x x x x x x x
2
2
1
3
1
*9
2
2 4. 1
3
31
2
2 4. 1
31
m
m
m
m
m
m
m
m
m









.
----The End----
October 4, 2021
[NGUYN THÀNH NHÂN TRƯỜNG ĐI HC AN GIANG]
56
Nghiên cu đnh lý Viète và ng dng | mathvn.com
| 1/56

Preview text:

October 4, 2021 [NGUYỄN THÀNH NHÂN – TRƯỜNG ĐẠI HỌC AN GIANG]
ỨNG DỤNG ĐỊNH LÝ VIÈTE ĐẠI SỐ SƠ CẤP A. LỊCH SỬ.
François Viète, Seigneur de la Bigotière ( tiếng Latinh : Franciscus Vieta ; 1940 - 23
tháng 2 năm 1603) là một nhà toán học người Pháp có công trình về đại số mới là một
bước tiến quan trọng đối với đại số hiện đại, do việc sử dụng sáng tạo các chữ cái làm
tham số trong phương trình và đồng thời ứng dụng chúng trong việc biến đổi và giải
phương trình. Ông là một luật sư về thương mại, và từng là ủy viên hội đồng bí mật cho
cả Henry III và Henry IV của Pháp.
Ông đã phát hiện ra mối liên hệ giữa các nghiệm và các hệ số của phương trình. Ông còn
là một chuyên gia về giải các mật mã trong thế chiến giữa Pháp và Tây Ban Nha. Ông mất năm 1603.
Thành tựu nổi bật: Đại số mới. Nền
Vào cuối thế kỷ 16, toán học được đặt dưới sự bảo trợ kép của người Hy Lạp, họ đã
mượn các công cụ của hình học và người Ả Rập, những người cung cấp các thủ tục cho
phép giải. Vào thời của Viète, đại số do đó dao động giữa số học, điều này làm xuất hiện
một danh sách các quy tắc và hình học có vẻ chặt chẽ hơn.
Đại số biểu tượng của Viète
Viète đã tạo ra nhiều đổi mới: công thức nhị thức , sẽ được Pascal và Newton lấy, và
các hệ số của đa thức thành tổng và tích các gốc của nó , được gọi là công thức Viète .
Đại số hình học
Viète rất thành thạo trong hầu hết các công cụ hiện đại, nhằm mục đích đơn giản hóa các
phương trình bằng cách thay thế các đại lượng mới có mối liên hệ nhất định với các đại
lượng chưa biết ban đầu. Một tác phẩm khác của ông, Recensio canonica effectionum
learningarum
, mang dấu ấn hiện đại, sau này được gọi là hình học đại số — một bộ sưu
tập các giới thiệu cách xây dựng các biểu thức đại số chỉ với việc sử dụng thước và compass.
1 Nghiên cứu định lý Viète và ứng dụng | ▫▪ mathvn.com
October 4, 2021 [NGUYỄN THÀNH NHÂN – TRƯỜNG ĐẠI HỌC AN GIANG] B. ĐỊNH LÝ VIÈTE.
Trong toán học, định lý Viète hay công thức Viète (có khi viết theo phiên âm tiếng
Việt là Vi-ét), do nhà toán học Pháp François Viète tìm ra, nêu lên mối quan hệ giữa
các nghiệm của một phương trình đa thức (trong trường số phức) và các hệ số của nó.
I. Định lý Viète cho phương trình bậc hai.
1. Bài toán mở đầu. Xét phương trình bậ 2
c hai: y ax bx c a  0   1 . 2
Giả sử:   b  4ac  0 .  bx    1  2a Ta có: 
là hai nghiệm tổng quát của phương trình 2
ax bx c  0 .  b x     2  2a
b   b   b
S x x    1 2  2a 2a a Khi đó:  2 . b            2 2 b  4ac b b b    . c P x x     1 2 2 2  2a 2a 4a 4a a 2. Định lý Viète. Extra Techniques Định lý Viète Nếu x , x 1
2 là hai nghiệm (trên trường số phức
, có thể nghiệm đơn hoặc nghiệm kép)  b
x x S    1 2  a của phương trình: 2
ax bx c  0 , thì:  . cx x P  1 2  a Chứng minh:
2 Nghiên cứu định lý Viète và ứng dụng | ▫▪ mathvn.com
October 4, 2021 [NGUYỄN THÀNH NHÂN – TRƯỜNG ĐẠI HỌC AN GIANG]
Giả sử: x , x
ax bx c  1
2 là hai nghiệm của phương trình 2 0 .
Khi đó, phương trình bậc hai  
1 tương đương với phương trình y ax x x x 1   2  . Như vậy, ta có đẳ 2
ng thức: ax bx c a x x x x 1   2  . 2 2
Hay: ax bx c ax a x x x ax x 1 2  1 2 .  b     x x b a x x    1 2  1 2  Đồ a
ng nhất hệ số hai vế, ta thu được:    (đpcm). c ax x c  1 2 x x  1 2  a
Như vậy, một câu hỏi được đặt ra: Liệu rằng có hay không một Định lý Viète tổng quát
trên trường số thực cho một đa thức có bậc n ?
Câu trả lời là có và xin được trình bày tiếp ở phần dưới đây.
II. Định lý Viète cho phương trình đa thức bất kỳ.
1. Bài toán mở đầu.
Xét phương trình bậc n theo ẩn x tổng quát như sau: n n 1
y a x a x   ...  a x a , a  0 2 n n 1  1 0  n    n n
Giả sử: x , i  1, n
a x a x  ...  a x a  0 i
n nghiệm của phương trình 1 n n 1  1 0 .
Khi đó, phương trình bậc n tương đương với phương trình:
y a x x x x ... x x n 1   2   n  Như vậ n n 1 
y, ta có: a x a x
 ... a x a a x x
x x ... x x n n 1  1 0 n  1   2   n n n 1
a x a x   ... n
a x a a x n n 1  1 0 n   a x x ...x     n 1 2 n      n elements
3 Nghiên cứu định lý Viète và ứng dụng | ▫▪ mathvn.com
October 4, 2021 [NGUYỄN THÀNH NHÂN – TRƯỜNG ĐẠI HỌC AN GIANG]     a x x x x ... x x ... x x        n 1 2 1 3 k k 1  n 1  n    nn  1  elements  2 
.......................................................................     n 1
1  a x x ...x x x ...x x  ...  x x ...x n 1 2 n 1  1 2 n2 n 2 3 n      n elements     1 n a x x ...x n 1 2 n
Đồng nhất hệ số ở hai vế, ta thu được Định lý Viète mở rộng như sau: Extra Techniques
Định lý Viète mở rộng Nếu x , i 1, , n n    i
 là hai nghiệm (trên trường số phức , có thể nghiệm đơn hoặc n n
nghiệm kép) của phương trình: 1 a x a x
 ...  a x a  0 n n 1  1 0 , thì: nan 1
 x x x  ...  x    i 1 2 n i 1  ann an2
  x x x x x x x x  .... x x i j 1 2 1 3 2 3 n 1  n 1
 ijn an
........................................   .................... .  n    a x x ...x
x x ...x x x ...x x  ...  x x ...x  1 n  . 2 3 n   1 1 1 i 2 i n i 1 1 2 n 1  1 2 n2 n 1    a 1 i  2i ...   ni 1nnn a
x x x ...x   i n  n 0 1 . 1 2  i 1 an
Lưu ý: Trong mỗi hàng k bất kỳ, vế trái của đẳng thức là tổng của các tích từng cụm k
các nghiệm của phương trình trên. Và vế phải của đẳng thức được tính một cách tổng a
quát theo công thức:   1 k nk a . n
4 Nghiên cứu định lý Viète và ứng dụng | ▫▪ mathvn.com
October 4, 2021 [NGUYỄN THÀNH NHÂN – TRƯỜNG ĐẠI HỌC AN GIANG]
Một số tổng quát thường gặp:
Phương trình bậc ba:
Nếu x , x , x
ax bx cx d  1 2
3 là nghiệm của phương trình: 3 2 0 thì công thức Viète  b
x x x    1 2 3 a   c
cho ta: x x x x x x  1 2 2 3 3 1 a .   d x x x    1 2 3  a
Hệ quả 1 của định lý Viète: n n 1 
Giả sử phương trình: a x a x
 ...  a x a  0 3 a i n n n 1  1 0
  có các hệ số , 0, i thỏa mãn: n  n 1            i 2 2
a   a  0 x  3 2k 2k 1  khi và chỉ khi:
1 là một nghiệm của phương trình  . k0 k 1  n  n 1            ii 2 2
a   a  0 x   3 2k 2k 1  khi và chỉ khi:
1 là một nghiệm của phương trình  . k0 k 1  Chứng minh i: n n 1   2  2     
Giả sử:  a   a
 0  a  a a  ...  a 2k 2k 1  0 n n 1  1 . k0 k 1  Khi đó: 3   n
a x a a x   a
  a x a n n   n 1 . . 0 n 1  n 1    1 1 ;
a x   a
x     a x   n n
1 n  n 1 1 ... 1 0 1  1    x  
1 a x   x     a
x   x      a x   a   n   n 1 n 2 ...
1 n  n 2 n 3 ... 1 ... 1 0 1  2   1
5 Nghiên cứu định lý Viète và ứng dụng | ▫▪ mathvn.com
October 4, 2021 [NGUYỄN THÀNH NHÂN – TRƯỜNG ĐẠI HỌC AN GIANG]
x 1 là một nghiệm. i :
Giả sử: x  1 là một nghiệm của phương trình 3, khi đó, ta có: n  n 1           aa    a
a   a a   a a  a   an   1 n n   1 n ...   2 2 1 1 0 ... 0 1 1 0 n n 1  1 0 2k 2k 1  . k0 k 1 
Vậy: i hoàn toàn được chứng minh. ii: Bài toán phụ:  2 1
x  1  x   1   2 2 1
x x   ...    1 k t t t k
x  ...  1  x  1. 4 2t 1    Ta có:  * kx 1    kx2 2 1   k x   1  kx  
1  ...  x   1 . 5 2t  
Một điều đáng nói, ở đây k hoặc chẵn hoặc lẻ, nhưng điều đó không quan trọng vì nếu k  2 k
thì ta xét x  
1 như ở đẳng thức 5 , nếu k  2 1 thì hoàn toàn rút được
lượng nhân tử x  
1 theo đẳng thức 4. Như vậy dù k là bội  nào đó của 2 , thì đến
một số đủ lớn các bước (sau  bước), ta sẽ thu được nhân tử  x   1 . n n 1   n n 1   2  2  2  2         
Giả sử:  a   a
 0  a  a   a 2k 2k 1  0 2k 2k 1  . k0 k 1  k 1  k 1  n 1  n Khi đó: 3  n a x a  a x a       
  a x a a x a n   n 1 1 n n  1 ... n      2 0 1 1 2 2   1 1   .  x  
1 a   a   . . a x 1  a   0  x  1   3 n n n 1  n 1  2   1
là một nghiệm của  .
ii : Giả sử: x  1
 là một nghiệm của phương trình 3, khi đó, ta có:
6 Nghiên cứu định lý Viète và ứng dụng | ▫▪ mathvn.com
October 4, 2021 [NGUYỄN THÀNH NHÂN – TRƯỜNG ĐẠI HỌC AN GIANG] a   a  
  a   a    a    a   
a a a n
1n n  n 1 1 ...  1 0
 1n n  n 1 1 ... 1 k ... 0 1 1 0 n 1    k 1 0 n n 1    . 2   2     
 a   a  0 2k 2k 1  k0 k 1 
Vậy: ii hoàn toàn được chứng minh. Extra Techniques Study tips Xét phương trình: 2
ax bx c  0 a  0 6 . x  1 1 
Nếu phương trình 6 có tổng a b c  0 thì: 
c là hai nghiệm của 6 . x  2  ax  1  1 
Nếu phương trình 6 có tổng a b c  0 thì: 
c là hai nghiệm của 6 . x   2  a
Hệ quả 2 của định lý Viète: n n 1 
Giả sử phương trình: a x a x
 ...  a x a  0 3 x i n n n 1  1 0
  có n nghiệm, kí hiệu , 1, i . nS  x  1 i i 1   nS   x x  2 i j
1ijn 
Nếu ta đặt: ........................   nS   x x ...x n 1  1 i 2 i n i 1  1 
 1i 2i... ni 1n   nS  x n i  i 1 
7 Nghiên cứu định lý Viète và ứng dụng | ▫▪ mathvn.com
October 4, 2021 [NGUYỄN THÀNH NHÂN – TRƯỜNG ĐẠI HỌC AN GIANG]
Khi đó: x ,i  1,n i
là nghiệm của phương trình:
x S x S x  . .    1 k S .x  . .       S x   S k  n 1 1 2 1 1 n n n n n k 0 7 1 2 n 1    n  . Chứng minh:
Theo định lý Viète mở rộng, ta suy ra:
Nếu x , i  1, n 3 i
là nghiệm của  , thì: nan 1
 x x x  ...  x    i 1 2 n i 1  ann an2
  x x x x x x x x  .... x x i j 1 2 1 3 2 3 n 1  n 1
 ijn an
........................................   ....................  n    a x x ...x
x x ...x x x ...x x  ...  x x ...x  1 n  . 2 3 n   1 1 1 i 2 i n i 1 1 2 n 1  1 2 n2 n 1    a 1 i  2i ...   ni 1nnn a
x x x ...x   i n  n 0 1 . 1 2  i 1 anan 1 S    1 anan2 S  2  an
Như vậy, dễ dàng ta có: ..................  .   aS   n  n 1 1 1 . 1  anaS   n  n 0 1 .  ana a a a n n 1  n 1  n2 n2
Vậy phương trình 7 tương đương với: 1 0 x x x  . .  x   0 a a a a . n n n n
8 Nghiên cứu định lý Viète và ứng dụng | ▫▪ mathvn.com
October 4, 2021 [NGUYỄN THÀNH NHÂN – TRƯỜNG ĐẠI HỌC AN GIANG]
Vì: deg f x , x ,. ., x
n a  0 1 2 n  n .
Nhân cả hai vế cho a , ta thu được phương trình 3 n (đpcm). Extra Techniques Study tips
S x x Xét phương trình: 2
ax bx c  0 a  0 6 1 2
với   0 , nếu ta đặt:  . P x x  1 2 2 2 Thì: x , x
x Sx P  0, S  4P 1
2 là hai nghiệm của phương trình  .
C. MỘT SỐ TIPS GIẢI NHANH CÁC BÀI TOÁN ỨNG DỤNG ĐỊNH LÝ VIÈTE.
I. Dấu nghiệm của phương trình bậc hai. Dấu nghiệm x x
S x x P x x 1 2 1 2 1 2
Điều kiện cần Điều kiện đủ Trái dấu  ?  P  0 Cùng dấu     P  0   0 Cùng dương    
P  0, S  0 Cùng âm    
P  0, S  0
II. Một số đẳng thức cần lưu ý.
ix x  x x 2 2 2 2
 2x x S  2P 1 2 1 2 1 2
iix x x x x x 2 3 3 3x x        S.    2S 3P 1 2 1 2 1 2 1 2  
iiix x x x  2 2 2x x       2x x 2 4 4     2S 2P2 2  2P 1 2 1 2 1 2 1 2    iv 1 1 x x P 1 2    x x x x S 1 2 1 2
vx x  x x 2 2
 4x x S  4P x x 2 1 1 2 1 2  2 1
9 Nghiên cứu định lý Viète và ứng dụng | ▫▪ mathvn.com
October 4, 2021 [NGUYỄN THÀNH NHÂN – TRƯỜNG ĐẠI HỌC AN GIANG]
III. Ứng dụng đa thức đối xứng để giải quyết các bài tập áp dụng định lý Viète.
1. Định nghĩa. Giả sử A là một vành giao hoán có đơn vị, f x , x ,..., x 1 2
n  là một đa thức
của vành A x , x ,..., x  
f x , x ,..., x 1 2
n  . Đa thức  1 2
n  được gọi là một đa thức đối xứng của
n ẩn nếu f x ,x ,. .,x f x ,x ,. .,x 1 2 n
 1  2  n     
  với mọi phép thế  1 2 .... n
   1 2 .... n    
f x ,x ,. .,x
f x , x ,..., x
f x , x ,..., x x  1  2  n        suy ra từ  1 2
n  bằng cách thay trong  1 2 n  , 1
bởi x  ,. . , x x  1
n bởi n .
2. Định lý 1. Bộ phận gồm các đa thức đối xứng của vành A x , x ,..., x   1 2 n  là một vành
con của vành A x , x ,..., x   1 2 n  . Chứng minh:
Giả sử f x , x ,..., x
g x , x ,..., x 1 2 n  và  1 2
n  là những đa thức đối xứng của vành
A x , x ,..., x   1 2
n  , theo định nghĩa ta có:
f x ,x ,. .,x f x ,x ,. .,x 1 2 n
 1  2  n        Và
gx ,x ,. .,x g x ,x ,. .,x 1 2 n
 1  2  n         1 2 .... n
với mọi phép thế     1 2 .... n   . Thế thì:  
f x ,x ,. .,x g x ,x ,. .,x f x ,x ,. .,xg x , x ,. ., x 1 2 n   1 2 n
 1  2  n   1  2  n              ,
f x ,x ,. .,x .g x ,x ,. .,x f x ,x ,. .,x
.g x , x ,. ., x 1 2 n   1 2 n
 1  2  n   1  2  n             
10 Nghiên cứu định lý Viète và ứng dụng | ▫▪ mathvn.com
October 4, 2021 [NGUYỄN THÀNH NHÂN – TRƯỜNG ĐẠI HỌC AN GIANG]
với mọi phép thế  . Từ đó, suy ra bộ phận gồm các đa thức đối xứng của vành
A x , x ,..., x  
A x , x ,..., x  1 2
n  là một vành con của vành  1 2 n  .
Chú ý: Có thể coi mỗi phần tử của vành A là một đa thức đối xứng đặc biệt. Thật vậy, aA 0 0 0
thì ta có thể viết: a x x ...x 1 2 n .
Các đa thức đối xứng cơ bản: n
  x x ... x   x 1 1 2 n i i 1   n
 x x x x x x .... x x    x x 2 1 2 1 3 2 3 n 1  n i j
1ijn
........................................   ....................  n  
x x ...x x x ...x x  ...  x x ...x   x x ...x n 1  1 2 n 1  1 2 n2 n 2 3 n 1 i 2 i n i 1  1   1i 2i ...
  ni1nn  
x x ...x   x n 1 2 n i  i 1 
Theo Định lý 1 thì mọi đa thức của các đa thức đối xứng cơ bản  ,  ,..., 1 2 n cũng là
một đa thức đối xứng của n ẩn x , x ,. ., x 1 2
n . Chiều ngược lại cũng đúng, đó chính là nội
dung của định lý cơ bản về đa thức đối xứng dựa trên các Bổ đề sau. a a a
3. Bổ đề 1. Giả sử f x , x ,..., xx x x ... na x 1 2
n  là một đa thức đối xứng khác 0 và 1 2 3 1 2 3 n
là hạng tử cao nhất của nó, thế thì: a a  ...  a 1 2 n . Chứng minh:
Ta phải chứng minh: aa , i   2,n f x ,..., x i 1  i . Vì  1
n  là một đa thức đối xứng nên nếu thay x x x x i 1 , ta đượ
 bởi i và hoán vị ngược lại thay i bởi i 1  c: 1 a i a 1  x ...  i a x x ... na x 1 i i 1  n
cũng là một hạng tử của f x ,..., x 1 n  .
Giả sử: a a i i 1 , khi đó: 
11 Nghiên cứu định lý Viète và ứng dụng | ▫▪ mathvn.com
October 4, 2021 [NGUYỄN THÀNH NHÂN – TRƯỜNG ĐẠI HỌC AN GIANG]
a ,...,a ,a ,a ,...,a a ,...,a ,a ,a ,...,a 1 i2 i i 1  n   1 i2 i 1  i n  . Điề a a a a
u này mâu thuẫn với giả thuyết 1 i 1  x ...  i x x ... n x 1 i 1  i n là hạng tử cao nhất.
4. Bổ đề 2. Giả sử a ,. ., a 1
n là những số tự nhiên sao cho:
a a  ...  a 1 2 n a a a a aa a
thế thì đa thức f x , x ,. ., x    . . nnn   1 2 n  1 2 2 3 1 2 1 2 n 1  n trong đó  ,..., 1
n là các đa thức đối xứng cơ bản, có hạng tử cao nhất là 1 a 2 a x x ... na x 1 2 n . Chứng minh:
Các hạng tử cao nhất của  , ,. ., , 1 2 n 1 
n theo thứ tự là:
x , x x ,. ., x x . .x , x x . .x 1 1 2 1 2 n 1  1 2 n
Ta có hạng tử cao nhất của f x ,..., x 1 n  là: a a x
x x  2a 3a . .x x . .x    x x xx x x n  n a 1 a a 1 2 n  . . n n 1 a 2 a . . na 1 1 2 1 2 1 1 2 1 2 n (đpcm).
Sở dĩ ta kết luận được hạng tử như thế là do ta chứng minh được một Định lý sau:
Định lý * . Giả sử f x , x ,..., x
g x , x ,..., x 1 2 n  và  1 2
n  là hai đa thức khác không của vành
A x , x ,..., x a a b b  11 c x ... n x 11 d x ... n x c d  0 1 2
n  có hạng tử cao nhất theo thứ tự là 1 1 1 n và 1 1 1 n . Nếu 1 1 a b a b
thì hạng tử cao nhất của đa thức tích f x , x ,..., x .g x , x ,..., x 11 11 c d x ... n n x  1 2 n   1 2 n  là 1 1 1 1 1 n . Chứng minh: f
  x , x ,..., a
x c x ... a n x  ... l ac x ... a x 1 2 n  11 1 1 ln 1 1 n l 1 n Giả sử: 
đã được sắp xếp theo lối từ điển. g
  x , x ,..., b
x d x ... b n x  ... m bd x ... m b x 1 2 n  11 1 1 n 1 1 n m 1 n Điều đó có nghĩa là:
12 Nghiên cứu định lý Viète và ứng dụng | ▫▪ mathvn.com
October 4, 2021 [NGUYỄN THÀNH NHÂN – TRƯỜNG ĐẠI HỌC AN GIANG]
a ,. .,a a ,. .,a , i   2,l 11 1n   1i in Và
b ,. .,b b ,. .,b , i   2,m 11 1n   1i in . Ta sẽ chứng minh: 11 a  11 b 1 a 1 c d x ... n b n x  1 1 1 n
là hạng tử cao nhất của đa thức tích f x ,..., x .g x ,..., x 1 n   1 n  .
Nhân f x ,..., x g x ,..., x 1 n  với  1 n  , ta được:
f x ,...,x g x xc d x x  
i l j m n .  ,..., n i
a 1 bj1 ... ian bjn , 1, ; 1, 1 1 i j 1 n  . i, j a b a b Mỗi hạng tử i1 j1 c d x ... in jn x a b ,. ., n a b i j 1 n
cho ta phần tử  1i j1 in jn  . Nhưng vì, ta lại có:
+) Nếu a ,...,a b ,...,b
a c ,...,a c b c ,...,b c , c ,..., n c  1 n  1 n   1 n  thì  1 1 n n   1 1 n n  .
Thật vậy! Vì: a ,...,a b ,...,b i n 1 n   1
n  nên có một chỉ số 1, sao cho:
a b ,. .,a b ,a b 1 1 i 1  i 1  i i .
Do đó: a c b c ,. .,a c b c ,a c b c 1 1 1 1 i 1  i 1  i 1  i 1  i i i i (đpcm).
+) Nếu a ,...,a b ,...,b
c ,...,c d ,...,d 1 n   1 n  và  1 n   1 n  , thì:
a c ,...,a c b d ,...,b d 1 1 n n   1 1 n n  . Thật vậy!
Ta có: a c ,...,a c b c ,...,b c b d ,...,b d 1 1 n n   1 1 n n   1 1 n n (đpcm).
Do vậy, ta có các bất đẳng thức sau:
13 Nghiên cứu định lý Viète và ứng dụng | ▫▪ mathvn.com
October 4, 2021 [NGUYỄN THÀNH NHÂN – TRƯỜNG ĐẠI HỌC AN GIANG]
a b ,...,a b a b ,...,a b , j 2,m 11 11 1n 1n
 11 j1 1n jn
a b ,...,a b a b ,...,a b , i  2,l 11 11 1n 1n   1i 11 in 1n
 a b ,...,a b a b ,...,a b , i  2,l, j  2,m 11 11 1n 1n
 1i j1 in jna b a b Vậy hạng tử 11 11 1 1 c d x ... n n x  1 1 1 n
chính là hạng tử cao nhất của đa thức tích. 5. Bổ đề 3.
Giả sử g  ,..., 1
n  là một đa thức của các đa thức đối xứng cơ bản g ,. ., a
  c  . . a n   . . m ac  . . m a n  1 n  11 1 1 1 1 n m 1 n
trong đó c  0, i  1,m
a ,. .,a a ,. .,a , i j i , và  1i in   j1 jn .
Thế thì: g  ,...,  0 1 n  . Chứng minh: Trong g  ,..., 
x x  . .  x ,. .,
x x  ...  x 1 n  , thay 1 bằng 1 2 n n bằng 1 2
n ta được một đa
thức của các ẩn x , x ,. ., x 1 2 n . gm
x x  ...  x ,..., x x ...x f x ,..., x   f x ,...,x 1 2 n 1 2 n   1 n i  1 n i 1  với
f x ,...,x c x x   x x x x i m n   ... n i a 1 ... ... i a n , 1, 1 1 1 1 2  1 2 3 .
Hạng tử cao nhất của đa thức f x ,..., x * 1  1
n  theo Định lý   là: a a 11 a c x x x ... x x ... n i k i k xc x x ... km x 1 1
 1 2 12  1 2 n 1 1 2 i 1 2 k
14 Nghiên cứu định lý Viète và ứng dụng | ▫▪ mathvn.com
October 4, 2021 [NGUYỄN THÀNH NHÂN – TRƯỜNG ĐẠI HỌC AN GIANG]
a a  ...a k 1 i i2 in 1 i  
a  ...a k với: i2 in i2  .
.................................  a kin in
Hạng tử cao nhất của mỗi đa thức f x ,..., x k ,k ,..., n k i 1
n  , cho ta phần tử  1 i i2 in  .
Ta có: k ,k . .,k
k ,k ,. .,k , i j 1 i i2 in
j1 j2 jn .
Vì nếu: k ,k . .,k
k ,k ,. .,k , i j 1 i i2 in
j1 j2 jn , thì:
a k k k k a 1 i 1 i i2 j1 j2 j1
a k k k k a i2 i2 i3 j2 j3 j2
.................................................
a k k a in in jn jn
với i j , mâu thuẫn với giả thuyết. n
sắp thứ tự toàn phần nên bộ phận hữu hạn gồm các phần tử k ,k ...,k i m 1 i i2 in  với 1, k k
có phần tử lớn nhất, chẳng hạn k ,k ,...,k 11 c x ... n x 11 12
1n  là phần tử lớn nhất. Do đó 1 1 1 n là hạng
tử cao nhất của f x ,..., x 1 n  .
Vậy: g  ,...,
f x ,..., x  0 1 n   1 n (đpcm).
Hệ quả. Giả sử:
hx ,x ,. ., a
x c x . . a n x  . . m ac x . . m a n x 1 2 n  11 1 1 1 1 n m 1 n
h'x ,x ,. ., a
x c x . . a n x  . . m ac x . . m a n x 1 2 n  ' 11 1 ' 1 1 1 n 1m 1 n
là hai đa thức trong đó a ,. .,a a ,. .,a  1 i in
j1 jn khi i j, sao cho:
15 Nghiên cứu định lý Viète và ứng dụng | ▫▪ mathvn.com
October 4, 2021 [NGUYỄN THÀNH NHÂN – TRƯỜNG ĐẠI HỌC AN GIANG]
h ,...,  h'  ,..., 1 n   1 n
Thế thì c c ', i  1, m i i . Chứng minh:
Giả sử có c c ' i i .
Đặt: g ,...,  h  ,...,  h'  ,..., 1 n   1 n   1 n  . a a a a
 c c '  . . n
 . .  c c ' m  . . mn  1 1  11 1 1 nm m  1 1 n Vì: c c ' c c '  0 1 1 , nên 1 1 .
Theo Bổ đề 3, ta có: g  ,...,  0 1 n  .
Nhưng theo giả thuyết thì: g ,...,  0 1 n  , mâu thuẫn.
6. Định lý 2. (Định lý cơ bản về đa thức đối xứng).
Giả sử f x , x ,..., x A x , x ,..., x  1 2 n   1 2
n  là một đa thức đối xứng khác không, khi đó có một
và chỉ một đa thức h x , x ,..., x A x , x ,..., x
f x , x ,..., x h  , ,..., 1 2 n   1 2 n  sao cho  1 2 n   1 2 n  . Trong đó  , ,. ., 1 2
n là các đa thức đối xứng cơ bản. Chứng minh: Sự tồn tại. a a
Ta hãy sắp xếp f x , x ,..., xx x ... na x 1 2
n  theo lối từ điển, giả sử 1 2 1 2 n
là hạng tử cao nhất của
f x ,x ,...,x 1 2
n  . Theo Bổ đề 1, ta có:
a a  ...  a 1 2 n
Mặt khác, theo Bổ đề 2, thì đa thức:
16 Nghiên cứu định lý Viète và ứng dụng | ▫▪ mathvn.com
October 4, 2021 [NGUYỄN THÀNH NHÂN – TRƯỜNG ĐẠI HỌC AN GIANG] 1 a  2 a 2 a  3 a a 1   ... n  n a n a   1 2 n 1  n cũng có hạ a a ng tử cao nhất là: 1 2  x x ... na x 1 2 n . Xét hiệu:
f x ,. ., x f x ,. ., a a a a x   . . n a   n a n a   1  1 n   1 n  1 2 2 3 1 1 2 n 1  n .
Nếu f x ,..., x  0 1  1 n
, thì ta sắp xếp nó theo lối từ điển và giả sử 1 b 2 bx x ... nb x 1 2 n
là hạng tử cao nhất của nó.
Theo Định lý 1 thì f x , x ,..., x 1  1 2
n  cũng là một đa thức đối xứng, và do đó ta có:
b b  ...  b 1 2 n .
Mặt khác, từ biểu thức của hiệu hai đa thức, ta có:
a ,...,a b ,...,b 1 n   1 n  Do đó: a b 1 1 . b b b b b b b Xét hiệu: f
x ,. ., x f x ,. ., x    . . n  n n   2  1 n  1  1 2  1 2 2 3 1 1 2 n 1  n . Nếu f x ,..., x  0 2  1 n
, ta hãy sắp xếp nó theo lối từ điển và giả sử 1 cx ... nc x 1 n
là hạng tử cao nhất của nó. Cũng lý luận tương tự đối với f x ,..., x , ta đượ 1  1 n  c:
c c  ...  c 1 2 n Với
17 Nghiên cứu định lý Viète và ứng dụng | ▫▪ mathvn.com
October 4, 2021 [NGUYỄN THÀNH NHÂN – TRƯỜNG ĐẠI HỌC AN GIANG]
b ,b ,...,b c ,c ,...,c 1 2 n   1 2 n  .
Ta nhận thấy rằng dãy a ,...,a b ,...,b c ,...,c  ... 1 n   1 n
 1 n không thể giảm vô hạn, tức là
quá trình lập luận trên diễn ra không thể vô tận. Sau một số hữu hạn bước, ta sẽ có: 0  f x x
x          k  , ,. ., l l l l . . n l n l n l 1 2 n  1 2 2 3 1 1 2 n 1  n
Vậy, từ các kết quả trên, ta có:
f x ,x ,. .,x                    n  1 a 2 a 2 a 3 a n a 1 b 2 b 2 b 3 b n b 1 l 2l 2 l 3 . . . . . . l . . nl 1 2 1 2 n 1 2 n 1 2 n .
Vậy đa thức h x , x ,..., x 1 2
n  cần tìm là đa thức:
hx ,x ,. .,xx x xx x xx x      x n  1 a 2 a 2 a 3 a n a 1 b 2 b 2 b 3 b n b 1 l 2l 2 l 3 . . . . . . l . . nl 1 2 1 2 n 1 2 n 1 2 n Tính duy nhất.
Giả sử có một đa thức h ' x ,..., x
h'  ,...,  f x ,..., x 1 n  sao cho  1 n   1 n  .
Thế thì: h ' ,...,  h  ,..., 1 n   1 n  .
Áp dụng hệ quả của Bổ đề 3 ta có:
hx ,...,x h' x ,...,x 1 n   1 n  . n n
Hệ quả. Giả sử f x 1  x a x  ...  a 1
n là một đa thức bậc n trên trường K , có n nghiệm  , ,. .,
g x , x ,..., x 1 2
n trong trường E nào đó chứa K như một trường con và giả sử  1 2 n
K x , x ,..., x  
g  , ,...,  K 1 2
n  là đa thức đối xứng. Khi đó:  1 2 n  . Chứng minh:
Thật vậy! Theo định lý cơ bản về đa thức đối xứng, tồn tại   K x , x ,..., x   1 2 n  sao cho
gx ,x ,...,x    , ,..., 1 2 n   1 2 n  .
Mặt khác, theo công thức Viète, ta có:
18 Nghiên cứu định lý Viète và ứng dụng | ▫▪ mathvn.com
October 4, 2021 [NGUYỄN THÀNH NHÂN – TRƯỜNG ĐẠI HỌC AN GIANG]
  , ,...,  1 ka K k 1 2 n    k . Bởi vậy
g , ,. .,     ,. ., ,. .,  ,. .,  a ,a ,. ., 1 na K 1 2 n
 1 1 nn 1 n  1 2   n .
Phép chứng minh Định lý 2 cho phép chúng ta biết cách biễu diễn một đa thức đối xứng qua
các đa thức đối xứng cơ bản. Trong thực tế để việc biểu diễn nhanh chóng hơn, chúng ta có
nhận xét rằng đa thức đối xứng f x , x ,..., x 1 2
n  có thể không phải là đẳng cấp, nhưng các hạng
tử có cùng một cấp của nó lập thành một đa thức đối xứng đẳng cấp, do đó f x , x ,..., x 1 2 n  là
tổng của những đa thức đối xứng đẳng cấp.
Bây giờ giả sử f x , x ,..., x A x , x ,..., x  1 2 n   1 2
n  là đa thức đối xứng đẳng cấp bậc k và hạng tử cao nhất là 1 a 2 ax x ... na x 1 2 n .
Bậc của f x , x ,..., x 1 2 n  là
a a  ...  a k 1 2 n .
Các đa thức đối xứng cơ bản  ,..., 1
n có bậc theo thứ tự là 1,2,. ., n , nên đa thức tích 1 a  2 a 2 a  3 a a  ... n a  1 2 n
cũng là đẳng cấp và có bậc là:
a a  2 a a  ...  na a a  ...  a k 1 2  2 3 n 1 2 n .
Do đó theo Định lý 2, ta có:
f x ,x ,. .,x f x x x        n   , ,. ., n 1 a 2 a 2 a 3 a . . na 1 1 2 1 2 1 2 n
cũng là đẳng cấp bậc k nếu khác 0 .
Sắp xếp f x , x ,..., x 1  1 2
n  theo lối từ điển và giả sử hạng tử cao nhất là
19 Nghiên cứu định lý Viète và ứng dụng | ▫▪ mathvn.com
October 4, 2021 [NGUYỄN THÀNH NHÂN – TRƯỜNG ĐẠI HỌC AN GIANG] 1 b 2 bx x ... nb x 1 2 n
Thế thì a a  . .  a b b  . .  b k
a ,a ,...,a b ,b ,...,b 1 2 n 1 2 n và  1 2 n   1 2 n  .
Theo Định lý 2, ta có dãy hữu hạn
a ,a ,...,a b ,b ,...,b c ,c ,...,c  ... 8 1 2 n   1 2 n   1 2 n    trong đó
a a  ...  a 1 2 n
b b  ...  b 1 2 n
c c  ...  c 1 2 n .........................
a a  . .  a b b  . .  b c c  . .  c  . .  k 1 2 n 1 2 n 1 2 n .
Tập hợp các phần tử của dãy * là một bộ phận của tập hợp hữu hạn M  
t ,. .,t ,. ., t ,. .,t 11 1n   1m mn
Trong đó t t  ...  t
t t  . . t a a  . . a k 1 i i2 in và 1 i i2 in 1 2 n .
Vậy, theo Định lý 2 thì f m
x , x ,..., x        n it1 it2 it2 it3 ... itn 1 2 i 1 2 n . i 1 
các hệ số   A i
tìm được nhờ phương pháp hệ số bất định.
Chú ý: Nếu phần tử t ,...,t 8   1 i
in  không có mặt trong dãy   thì 0 i . Tập hợp M  
t ,. .,t ,. ., t ,. .,tn
f x , x ,..., x 11 1n   1m
 gọi là hệ thống số mũ của đa thức  1 2 n  .
20 Nghiên cứu định lý Viète và ứng dụng | ▫▪ mathvn.com
October 4, 2021 [NGUYỄN THÀNH NHÂN – TRƯỜNG ĐẠI HỌC AN GIANG]
D. MỘT SỐ ỨNG DỤNG CỦA ĐỊNH LÝ VIÈTE.
I. Một số ứng dụng.
Dạng 1. Tìm hai số khi biết tổng và tích.
Dạng 2. Tính giá trị biểu thức đối xứng.
Dạng 3. Tìm điều kiện của tham số để hai nghiệm liên hệ với nhau bởi một hệ thức cho trước.
Dạng 4. Tìm hệ thức liên hệ giữa các nghiệm độc lập với tham số.
Dạng 5. Thiết lập phương trình bậc hai.
Dạng 6. Xét dấu các nghiệm.
Dạng 7. Giải hệ phương trình đối xứng loại 1.
Dạng 8. Chứng minh bất đẳng thức.
Dạng 9. Ứng dụng trong bài toán cực trị.
Dạng 10. Ứng dụng trong bài toán tiếp tuyến.
Dạng 11. Ứng dụng hệ thức truy hồi.
Dạng 12. Ứng dụng tính các biểu thức lượng giác.
Dạng 13. So sánh nghiệm.
Dạng 14. Ứng dụng khác.
II. Bài tập áp dụng.
Dạng 1. Tìm hai số khi biết tổng và tích. S  2x
Câu 1. Tìm hai số a b khi biết tổng S và tích P :  . 2 2
P x y Giải Ta có: X  ,
a X b là nghiệm của phương trình: 2 2 2
X  2xX x y  0 .
21 Nghiên cứu định lý Viète và ứng dụng | ▫▪ mathvn.com
October 4, 2021 [NGUYỄN THÀNH NHÂN – TRƯỜNG ĐẠI HỌC AN GIANG] 2 2 2 2
Ta có:  '  x x y y  0 . Khi đó: 2
X x y x y . Vậy:  ; a b  
x  ;yx y,x  ;yx y.
Câu 2. Tìm hai số a b biết:
a b  9 a)  . 2 2
a b  41 Giải 2 2 81 a b 2 2 2 81 41
Ta có: a b  9  a b  81  a b   
 2ab  81 ab    20 2 2 . a b  9 Khi đó, ta có:  . ab  20 Như vậy, x  ,
a x b là hai nghiệm thực của phương trình: 2
x  9x  20  0 .  2 x  4
Ta có: x  9x  20  0   . x  5 Suy ra:  , a b    4;5,5;4. a b  5 b)  . ab  36 2 2 2
Ta có: a b  25  a b  4ab  25  a b  25  4ab  25  4.36  169 .
a b  13  Như vậy:  . ab  36 2 Suy ra: x  ,
a x b là hai nghiệm mỗi phương trình 2
x 13x  36  0 hoặc x 13x  36  0.
22 Nghiên cứu định lý Viète và ứng dụng | ▫▪ mathvn.com
October 4, 2021 [NGUYỄN THÀNH NHÂN – TRƯỜNG ĐẠI HỌC AN GIANG]x  4 +) Phương trình 2
x 13x  36  0 có hai nghiệm là:  . x  9 Do vậy:  , a b    4;9,9;4. x  4  +) Phương trình 2
x 13x  36  0 có hai nghiệm là:  . x  9  Do vậy:  , a b    9  ; 4  , 4  ; 9  .
Dạng 2. Tính giá trị biểu thức đối xứng.
Câu 1. Giả sử x , x , x
x px q  1 2
3 lần lượt là ba nghiệm của phương trình 3 0 . 2 2 2
Tính giá trị của biểu thức: S   x x x x x x 1 2   2 3   1 3  . Giải 2 2 2
Đặt: gx , x , x x x x x x x 1 2 3 
 1 2  1 3  2 3 2 2 2
Dễ thấy: gx , x , x x x x x x x 1 2 3 
 1 2  1 3  2 3 là đa thức đối xứng.
Hệ thống số mũ: M    4;2;0;4;1  ;1 ;3;3;0;3;2  ;1 ;2;2;2.
Khi đó: gx , x , x  2 2 3 3 2
    a   b  c    d 1 2 3 1 2 1 3 2 1 2 3 3 . Chọn: x  1    0 1 1   +) x  1      1  b  4 2 2 . x 0     0  3  3    0 1 x  2 1  +)      3   d  27  2 . x x  1   2 3    2  3
23 Nghiên cứu định lý Viète và ứng dụng | ▫▪ mathvn.com
October 4, 2021 [NGUYỄN THÀNH NHÂN – TRƯỜNG ĐẠI HỌC AN GIANG]    3  1 x  1 1  +)      0  a  4  2 . x x  2   2 3    4  3    1  1 x  1 1  +)      1  c  18 2 .
x x  1  2 3    1  3
Khi đó: gx , x , x  2 2 3 3 2
    4   4 18    27 1 2 3 1 2 1 3 2 1 2 3 3 .    0 1 
Vì: x , x , x
x px q     p 1 2
3 lần lượt là ba nghiệm của phương trình 3 0 nên 2 .    q  3 Do đó, 3 2 S  4  p  27q .
Câu 2. Giả sử x, y, z là ba nghiệm của phương trình 3
  2021  2022  0 . Tính giá trị 4 4 4
của biểu thức S x y z ? Giải
x y z  0 
Theo định lý Viète, ta có: xy yz xz  2021  . xyz  2022   Mặt khác, ta có:
x y z  0  x y z2 2 2 2
 0  x y z  2xy yz xz
 x y z 2  2
  xy yz xz 2 2 2 2  4 4 4 2 2 2 2 2 2
x y z  2x y  2y z  2x z  4 2 2 2 2 2 2 2 2 2
x y y z x z  2xy z  2xyz  2x yz 4 4 4
x y z  2 2 2 2 2 2 2 2 2 2
x y y z x z  2xy z  2xyz  2x yz  4 2 2 2
x yz xy z xyz  4 4 4
x y z  2xy yz xz2  xyzx y z  xy yz xz2 2 4 2  2.2021
24 Nghiên cứu định lý Viète và ứng dụng | ▫▪ mathvn.com
October 4, 2021 [NGUYỄN THÀNH NHÂN – TRƯỜNG ĐẠI HỌC AN GIANG]
Dạng 3. Tìm điều kiện của tham số để hai nghiệm liên hệ với nhau bởi một hệ thức cho trước.
Câu 1. Cho phương trình 2
y my p  0 có hai nghiệm là y y . Đị 1 và 2
nh m p để 1 1 1 và
cũng là nghiệm của phương trình này.  y 1 y 1 2 Giải Xét phương trình 2
y my p  0 * . Phương trình * 2 2
có nghiệm khi và chỉ khi:   m  4 p  0 hay m  4 p .
y y  m 1 2
Áp dụng hệ thức Viète, ta có:  . y y p  1 2  1 1 2  y y 1 2  2  m     1 y 1 y
1 y y y y p m 1 1 2  1 2 Khi đó, ta có: 1 2  .  1 1 1 .  1 y 1 y p m 1  1 2
 2  m  m   1 1 1  p m 1 Do ; * 1
cũng là nghiệm của phương trình   nên: .  y 1 y  1 1 2   p 2
 p m 1 Từ  
1 , 2, suy ra: p2  m  m 3 . +) Nếu m  2  . p 0  2  (vô lý). m
+) Nếu m  2  p  2 , ta đượ m , thay vào   c:  2  mmm
  mm m    m    2 2 1 . 1 4 2 2 .  m  2  m  2
25 Nghiên cứu định lý Viète và ứng dụng | ▫▪ mathvn.com
October 4, 2021 [NGUYỄN THÀNH NHÂN – TRƯỜNG ĐẠI HỌC AN GIANG]    m   m 1  1 2
m  2m  4  0   . m  1   5 Thử lại: 2
+) Với m  1, thì p  1
 thỏa điều kiện m  4p . 3 5 2
+) Với m  1 5 thì p   m  4p 2 thỏa điều kiện . 3 5 2
+) Với m  1 5 thì p   m  4p 2 thỏa điều kiện .      
Vậy các cặp m, p cần tìm là    3 5 3 5 1; 1 ,1 5; ,1 5,   2   2  .    
Câu 2. Cho phương trình 2 2
x  2mx m m  6  0 ( m là tham số).
1. Với giá trị nào của m thì phương trình đã cho có hai nghiệm x x 1 và 2 sao cho x x 18 1 2   x x 7 . 2 1
2. Với giá trị nào của m để phương trình đã cho có hai nghiệm x x 1 và 2 sao cho x x  8 1 2 . Giải
1. Để phương trình 2 2
x  2mx m m  6  0 có hai nghiệm thì: 2   m   2 '
m m  6  m  6  0  m  6    1 . Với điều kiện   1 , ta có: x x 18 x x 18 x x  2x x 18 1 2 1 2  2 2 2 1 2 1 2       , x x  0 1 2 x x 7 x x 7 x x 7 . 2 1 1 2 1 2
26 Nghiên cứu định lý Viète và ứng dụng | ▫▪ mathvn.com
October 4, 2021 [NGUYỄN THÀNH NHÂN – TRƯỜNG ĐẠI HỌC AN GIANG] 2 4m  2 2 m m  6 2 18 m m  6 9     , m  2,  m  3 2 2  . m m  6 7 m m  6 7 m  4  2 1
m  8m  48  0  
(thỏa điều kiện  
1 và đều khác 2 và 3 ). m  12  2
2. Với điều kiện   1 ,
x x  8  x x  2 x x  64  x x 2 2 2
 2x x  2 x x  64 2 1 2 1 2 1 2 1 2 1 2 1 2  . m  6    6   m  2  Nếu x x x x  0    3 1 và 2 cùng dấu thì: 1 2 2 m .  m  6  
m2m3     0 m  3
Khi đó: 2  x x 2 2
 64  4m  64  m  4  3 1 2
(thỏa điều kiện  ). 2 Nếu x x
x x  0 m m  6  m  2 m  3  0  2   m  3 4 1 và 2 trái dấu thì: 1 2     . 2
Khi đó: 2  x x  2
 4x x  64  4m  4 2
m m  6  64  m  10 1 2 1 2 
(không thỏa điều kiện 4 ).
Vậy để x x  8 m   1 2 thì: 4.
Dạng 4. Tìm hệ thức liên hệ giữa các nghiệm độc lập với tham số. 2
Câu 1. Giả sử phương trình mx  2m  3 x m  4  0 ( m là tham số) có hai nghiệm
thực phân biệt là x , x x , x 1
2 . Tìm hệ thức liên hệ giữa 1
2 không phụ thuộc vào m . Giải  2m  3 3  12 x x   2 
4 x x  8   1 2   1 2    Theo đị m m m nh lý Viète, ta có:    . m  4 4 12 x x 1 3     x x  3  1 2 1 2  m m  m
27 Nghiên cứu định lý Viète và ứng dụng | ▫▪ mathvn.com
October 4, 2021 [NGUYỄN THÀNH NHÂN – TRƯỜNG ĐẠI HỌC AN GIANG]
Suy ra: 4  x x  3x x  11  4x  4x  3x x  11 x , x 1 2  1 2 1 2 1 2
là hệ thức liên hệ giữa 1 2 ,
độc lập với tham số m . 2
Câu 2. Cho phương trình m  
1 x  2m  
1 x m  0.
a) Giải và biện luận phương trình.
b) Khi phương trình có hai nghiệm phân biệt x , x x , x 1
2 . Tìm một hệ thức liên hệ giữa 1 2 độc lập với m . Giải 1
a) Xét m  1, phương trình đã cho trở thành: 4
x 1  0  x  4 là nghiệm duy nhất. 2
Với m  1, ta có:  '  m  
1  mm   1  3m 1. 1
+) Với: m   , phương trình đã cho có hai nghiệ 3 m phân biệt. 1
+) Với m   , phương trình đã cho có mộ 3 t nghiệm kép. 1
+) Với m   , phương trình vô nghiệ 3 m.  2m   1 4 x x   2  1 2  m 1 m 1
b) Theo định lý Viète, ta có: 
lấy phương trình trên trừ đi  m 1 x x   1 1 2  m 1 m 1
4 lần phương trình dưới, ta có: x x  4x x  2  1 2 1 2 .
Vậy hệ thức liên hệ giữa x , x , độ
x x  4x x  2  1 2
c lập với tham số m là: 1 2 1 2 .
Dạng 5. Thiết lập phương trình bậc hai.
Câu 1. Tìm phương trình bậc hai có hai nghiệm x  2021 và x  2022 . Giải
28 Nghiên cứu định lý Viète và ứng dụng | ▫▪ mathvn.com
October 4, 2021 [NGUYỄN THÀNH NHÂN – TRƯỜNG ĐẠI HỌC AN GIANG]
S x x  4043 1 2 Ta có:  .
P x x  4086462  1 2
Như vậy, dễ dàng suy ra được, phương trình bậc hai có hai nghiệm x  2021 và x  2022 là: 2
x  4043x  4086462  0 .
Câu 2. Giả sử x , x
x px q  1
2 là hai nghiệm của phương trình 2 0 . Hãy lập một phương
trình bậc hai có hai nghiệm là x x x x 1 2 và 1 2 . Giải
x x  p Theo đị 1 2 nh lý Viète, ta có:  . x x q  1 2
X x x Bài toán đã cho đượ 1 1 2
c quy về việc tìm phương trình bậc hai nhận  làm nghiệm. X x x  2 1 2
S X X  p q Đặ 1 2 t:  .
P X X  pq  1 2
Như vậy! Phương trình bậ 2
c hai cần tìm là: x   p qx pq  0 .
Dạng 6. Xét dấu các nghiệm. x x
Câu 1. Cho hàm số C 2 3 3 : y x .  2
Tìm các giá trị của m để đường thẳng d : y mx m cắt đồ thị tại hai điểm thuộc về hai
nhánh của đồ thị C . Giải 2 x  3x  3
Phương trình hoành độ giao điểm của hai đồ thị d và C: mx m  * x .  2
   m   2 *
1 x  m 3 x  2m 3  0   1 .
29 Nghiên cứu định lý Viète và ứng dụng | ▫▪ mathvn.com
October 4, 2021 [NGUYỄN THÀNH NHÂN – TRƯỜNG ĐẠI HỌC AN GIANG]
Để d cắt C tại hai điểm thuộc hai nhánh khi và chỉ khi  
1 có hai nghiệm x , x 1 2 thỏa mãn x  2   x
x  2  0  x  2 1 2 hay 1 2 .
Đặt: t x  2, ta đưa  
1 về phương trình ẩn t : m   2
1 t  3m  
1 t 1  0 2 .
Phương trình 2 phải có hai nghiệm trái dấu. Khi đó: m   1 .  1  0  m  1.
Câu 2. Cho phương trình
m   3x  m   2 1 3
1 x x  4m 1  0   1 ( m là tham số).
Với giá trị nào của m , thì phương trình  
1 có ba nghiệm phân biệt, trong đó có hai nghiệm âm. Giải 3 2 Ta có: m  
1 x  3m  
1 x x  4m 1  0   1 .  m   3
1 x  m   2 2
1 x  4mx  4m x 1  0  m   2 1 x x   1  4m 2 x   1  x   1  0  m   2 1 x x  
1  4mx   1 x   1  x   1  0     x a
x 1 m   1 2  
1 x  4mx  4m 1  0     g
  x  m   2
1 x  4mx  4m 1  0 b Để phương trình  
1 có ba nghiệm thực phân biệt thì phương trình b phải có hai m  1  
nghiệm thực phân biệt khác 1, tương đương với:  '   m b 1 3 0 .  g    1  0
30 Nghiên cứu định lý Viète và ứng dụng | ▫▪ mathvn.com
October 4, 2021 [NGUYỄN THÀNH NHÂN – TRƯỜNG ĐẠI HỌC AN GIANG]m  1    1 1 Hay: m   m  1
 ,m  0,m  * 3 3 .  9m  0 
Với điều kiện * , phương trình  
1 có ba nghiệm phân biệt, trong đó có một nghiệm
x 1  0 và hai nghiệm còn lại x , x x x b . Do đó để 1 2  1
2  là nghiệm của     1 có ba
nghiệm phân biệt trong đó có hai nghiệm âm thì: x x  0 , tương đương vớ 1 2 i: m  1   4m 1  P x x   0 1    m  1  1 2  m 1   m      4   1 **. 4m  0 m S x x m  1            1 2  4  m 1  m  0 Để phương trình  
1 có ba nghiệm phân biệt, trong đó có hai nghiệm âm thì điều kiện cần 1 1 và đủ là: m  1  hoặc  m  4 3 .
Dạng 7. Giải hệ phương trình đối xứng.
x y y x  30
Câu 1. Giải hệ phương trình:  .
x x y y  35  Giải
u x  0 Đặt:  , hệ đã cho trở thành: v y  0  2 2   uv     u v u v uv   30 30    . 3 3
u v  35 uv
3 3uvuv  35
31 Nghiên cứu định lý Viète và ứng dụng | ▫▪ mathvn.com
October 4, 2021 [NGUYỄN THÀNH NHÂN – TRƯỜNG ĐẠI HỌC AN GIANG]
S u v 2 Tiếp theo ta đặt: 
, S  4P. P uv SP  30 SP  30 S  5
Ta thu được một hệ mới:      (thỏa mãn). 3 3
S  3SP  35 S  125 P  6
Theo định lý Viète, ta có: u,v là nghiệm của phương trình: 2
t  5t  6  0 . u  2  t  2 v  3 Khi đó:   . t 3   u  3  v  2
Dẫn đến nghiệm của hệ là  ; x y    4;9;9;4 2 2
x xy y  3 2
Câu 2. Giải hệ phương trình:  . 4 4
x y  5 Giải
Ta có: x y  x y 2 4 4 2 2 2 2  2x y . 2 2
u x y
u v  3  2 Đặt: 
, hệ đã cho trở thành:  . v xy 2 2
u  2v  5 u  3  v   2
Giải hệ đã cho, ta được:  II  . u  9  4 2  
v  6  3 2 2 2 2 2
x y  3 x y  3
Với hệ I  , thì:    . 2 2 xy   2 x y  2
32 Nghiên cứu định lý Viète và ứng dụng | ▫▪ mathvn.com
October 4, 2021 [NGUYỄN THÀNH NHÂN – TRƯỜNG ĐẠI HỌC AN GIANG]t  1 Theo đị 2 2
nh lý Viète, thì: x , y là nghiệm của phương trình: 2
t  3t  2  0 , ta được:  . t  2 2 x  1  2 y  2 xy  0 
Thế vào hệ, ta được:  . 2 x  2   2 y  1   xy  0  Suy ra nghiệm  ;
x y  1; 2; 1  ; 2 ; 2; 1  ;  2;1.
Trường hợp còn lại vô nghiệm.
Dạng 8. Chứng minh bất đẳng thức. 2
Câu 1. Cho x, y, z khác 0 , thỏa mãn: x y z xyz x yz . 2
Chứng minh rằng: x  3 . Giải 3
y z x x
Từ giả thuyết, ta có:  . 2 yz x Theo đị 2 3 2
nh lý Viète, thì y, z là nghiệm của phương trình: t  x xt x  0 .
Do tồn tại các số y, z , nên phương trình trên phải có nghiệm: 2 2 3 2 2  2 
Tức là:   0  x x  4x  0  x x   1  4  0  .      x 1  2
Vì: x  0 , nên: x   2 2 2 2 1  4  0    x  3  . 2   x 1  2 
33 Nghiên cứu định lý Viète và ứng dụng | ▫▪ mathvn.com
October 4, 2021 [NGUYỄN THÀNH NHÂN – TRƯỜNG ĐẠI HỌC AN GIANG]
x y z  5 7
Câu 2. Cho các số thực x, y, z thỏa mãn: 
. Chứng minh rằng: 1  x, y, z  .
xy yz xz  8 3 Giải
Từ giả thuyết, ta xem z là tham số, ta có hệ phương trình ẩn x, y :
x y  5  z
x y  5  z     xy z .  x y 8     xy  8  z  5z Theo đị 2
nh lý Viète thì x, y là nghiệm của phương trình: t  5  zt  8  z5  z  0 .
Do phương trình có nghiệm đối với x, y nên:
    z2    z  z 7 5 4 8 5
  0  1 z    3 .
Do vai trò bình đẳng của x, y,z nên ta có kết luận tương tự đối với x y .
Dạng 9. Ứng dụng trong bài toán cực trị.
Câu 1. Tìm tất cả các giá trị tham số m để đồ thị hàm số y f x 3
mx  m   2
2 x  1 mx  3
có hai điểm cực trị có hoành độ dương là? Giải
Ta có: f  x 2
 3mx  2m  2 x  1 m .
Để đồ thị hàm số có hai điểm cực trị có hoành độ dương thì:  
m  2  m  m 1 2 3 1  0   m  4   0      m   2 2 2  1  S  0    0
 2  m  0    m  0 . 3m 2    P  0  m  0  1 m    0     m  1 m
Câu 2. Tìm tất cả các giá trị nguyên của tham số m  1
 0;10 để hàm số:
34 Nghiên cứu định lý Viète và ứng dụng | ▫▪ mathvn.com
October 4, 2021 [NGUYỄN THÀNH NHÂN – TRƯỜNG ĐẠI HỌC AN GIANG] 3 y
x  4mx  m  2 3 2 1 x  1 2
có hai điểm cực trị x , x thỏa mãn x x x x là? 1 2 1 2 1 2 Giải 9 Ta có: y 
x  8mx  m  2 2 1 . 2
Hàm số y có hai điểm cực trị khi phương trình y  0 có hai nhgiệm phân biệt  
  m23m 18  9 . 2m  2 1 Theo đị 16m
nh lý Viéte ta có: x x  ; x x  . 1 2 1 2 9 9 2 16 m  5  2 6
x x x x  m  2 2 1 
m m 10m 1  0   . 1 2 1 2 9 9 m  5  2 6  m 1  0;01  0 .
Dạng 10. Ứng dụng trong bài toán tiếp tuyến. x  2
Câu 1. Cho hàm số y
có đồ thị C  và điểm A0;a . Hỏi có tất cả bao nhiêu giá x 1
trị nguyên của a để từ điểm A kẻ được hai tiếp tuyến đến C  sao cho hai tiếp điểm nằm
về hai phía của trục hoành? Giải 3  Ta có: y '   . x  2 1  x  2  Gọi tiếp điểm là 0 M x ;
 . Khi đó phương trình tiếp tuyến của C tại M là: 0 x 1  0   x
d : y f  x  x x  3 2  y x x  . 2  0  0 0 0 0 x   1 x 1 0 0
35 Nghiên cứu định lý Viète và ứng dụng | ▫▪ mathvn.com
October 4, 2021 [NGUYỄN THÀNH NHÂN – TRƯỜNG ĐẠI HỌC AN GIANG]   Vì đườ 3 x 2
ng thẳng d : y  x x
đi qua điểm A0;a . Khi đó: 2 0  0 x   1 x 1 0 0 3x x  2 0 0 2 2           x   a 3x x x 2 ax 2ax a 2 0 0 0 0 0 1 x 1 0 0  a   2
1 x  2 a  2 x a  2  0, x  1 1 0   0  0   
Từ A kẻ được 2 tiếp tuyến đến C   Phương trình  
1 có 2 nghiệm x phân biệt khác 1 0
  a  2 2  a   1 a  2  0
  3a  6  0         .     a
a 1 .1 2 a  2 2 .1 a  2  0  3   0
Khi đó phương trình (1) có hai nghiệm x , x  1. 1 2
Hai tiếp điểm nằm về hai phía của trục hoành a  2 2 a  2   2  4 x  2 x  2
x x  2 x x  4 1  2  1 2  1 2  a 1 a 1
y .y  0   0   0   0 1 2 x 1 x 1
x x x x 1 a  2 2 a  2 1  2  1 2  1 2     1 a 1 a 1
a  2  4a  8  4a  4  9a  6 2 a 1   0 
 0  3a  2  0  a   .
a  2  2a  4  a 1 3  3 a 1 x
Câu 2. Cho hàm số: C 1 : y  . Đườ
d y x m 2x ng thẳng :
. Với mọi m ta luôn có 1
d cắt C tại hai điểm phân biệt ,
A B . Gọi k ,k 1
2 lần lượt là hệ số góc của tiếp tuyến với
C tại ,AB. Tìm m để tổng k k 1
2 đạt giá trị lớn nhất. Giải
Phương trình hoành độ giao điểm của d và C là:  1 x 1 x
x m   2 2x . 1 g   x 2
 2x  2mx m 1  0*
x x  m 1 2 
Theo định lý Viète, ta có:  m 1 . x x   1 2  2
36 Nghiên cứu định lý Viète và ứng dụng | ▫▪ mathvn.com
October 4, 2021 [NGUYỄN THÀNH NHÂN – TRƯỜNG ĐẠI HỌC AN GIANG]
Giả sử: A x ; y ,B x ; y 1 1   2 2. 1  Ta có: y '   . 2x  2 1  1 k    1  2x  2 1 1
Nên tiếp tuyến của C  tại A B có hệ số góc lần lượt là:  . 1 k   2  2x   2 1 2 4 2 2 x x
 4 x x  2 1 1 1 2   1 2
Vậy: k k      1 2  . 2x  2 1 2x  2 1 4x x  2  x x  2 1 1 2 1 2 1 2 
4 x x 2 2x x   
 4x x   2 4    2 1 2 1 2 1 2 m m   1  4m  2
k k     1 2 4x x  2  x x  2 1  2
m  2  2m  2 1 1 2 1 2 
k k  4
m  8m  6  4
 m  2m  
1  2  4m  2 2 2 1  2  2 1 2
Vậy tổng k km   1
2 đạt giá trị lớn nhất bằng 2 khi 1.
Dạng 11. Ứng dụng hệ thức truy hồi.
Phương pháp truy hồi:
Xét phương trình bậc hai: 2 2
ax bx c  0, a  0. Giả sử x , x 1
2 là các nghiệm của nó. Đặ n n
t: S x x , n n 1 2 .
Lúc đó ta có hệ thức truy hồi tuyến tính sau: aS
bS cS  0 1 n2 n 1  n  .
37 Nghiên cứu định lý Viète và ứng dụng | ▫▪ mathvn.com
October 4, 2021 [NGUYỄN THÀNH NHÂN – TRƯỜNG ĐẠI HỌC AN GIANG] Chứng minh: n2 n2 n 1  n 1  n n Ta có: Sxxxx
x x x x x x n2 1 2  1 2  1 2 1 2 1 2 .     . b c S S   .S aS
bS cS  0 n2 n 1    n n2 n 1  n (đpcm).  a a
Câu 1. Tìm số nguyên lớn nhất không vượt quá   7 4 15 Giải x  4  15 Ta đặ 1 t:  . x  4  15  2 Khi đó: x , x x x   1
2 là nghiệm của phương trình: 2 8 1 0 . Đặ n n
t: S x x , n S
 8S S  0 n 1 2
. Ta có hệ thức: n2 n 1  n . S  8 1 S  62  2 S  488 3 
Ta tính được: S  3842 4 . S  30248  5 S  238142 6  S  1874888  7 Như vậ 7 7 7 y: x  1874888  x 0  x  1 1 2 . Mà 2 . 7
Suy ra: 1874887  x  18748888 1 .
Vậy số nguyên lớn nhất không vượt quá   7 4 15 là 1874887 .
Câu 2. Tìm chữ số tận cùng của phần nguyên của số   2021 5 3 3 .
38 Nghiên cứu định lý Viète và ứng dụng | ▫▪ mathvn.com
October 4, 2021 [NGUYỄN THÀNH NHÂN – TRƯỜNG ĐẠI HỌC AN GIANG] Giải x  53 3 Ta đặ 1 t:   x , x x x   1
2 là nghiệm của phương trình: 2 10 2 0 . x  5  3 3  2 Đặ n n
t: S x x , n n 1 2
. Theo hệ thức truy hồi, ta có: S  10S  2S n2 n 1  n .
Ta có: S  10  10 S 1 2k 1  . Để n n ý rằng: 1   x  0
S x S x S 1 2 , nên suy ra: n 1 n 2 n . n
Vậy nên x   S  10 S 1 n  . Vì 2021 là số lẻ nên 2021 .
Vậy chữ số tận cùng của phần nguyên của số   2021 5 3 3 là số 0 .
Dạng 12. Ứng dụng tính các biểu thức lượng giác.  9 9 17  17 3
Câu 1. Chứng minh rằng: cos cos  cos cos  cos cos   12 12 12 12 12 12 4 . Giải 3
Áp dụng công thức nhân ba, ta có: cos3x  4 cos x  3 cos x . 3       2 +) Với x   4cos  3cos   0 1 12 , ta có:      . 12  12  2  Nên: cos t t  
12 là nghiệm của phương trình: 3 2 4 3 0 2 . 9 17
Lập luận tương tự, ta có: cos cos 12 và
12 cũng là nghiệm của phương trình   1 .  9 9 17  17 3
Theo định lý Viète, ta có: cos cos  cos cos  cos cos   12 12 12 12 12 12 4 .
39 Nghiên cứu định lý Viète và ứng dụng | ▫▪ mathvn.com
October 4, 2021 [NGUYỄN THÀNH NHÂN – TRƯỜNG ĐẠI HỌC AN GIANG]
Câu 2. Cho b  0 , giả sử phương trình: 3 2
x ax x b  0
có ba nghiệm là x , x , x 1 2 3 . Chứng minh rằng:  1  1   1  1   1  1   x   x     x   x     x   x    4 1 2 2 3 3 1 x x x x x x .  1  2   2  3   3  1  Giải
Theo định lý Viète, ta có:
x x x x x x  1 1 2 2 3 3 1 .
Đặt: x  tan; x  tan ; x  tan  1 2 3 , thế thì ta có:
tan tan   tan  tan   tan tan  1. 
Suy ra:        k , k  
        kk   2 , suy ra: 2 2 2 , .
Đẳng thức cần chứng minh tương đương với:
tan cottan cot tan cot tan cottan cottan cot   4
Để ý rằng: tan x  cot x  2  cot 2x .
Vì thế, đẳng thức cần chứng minh trở thành:
cot 2 cot 2  cot 2 cot 2  cot 2 cot 2 1.
Đẳng thức hiển nhiên đúng, vì: 2  2  2    k , k   (đpcm).  1  1   1  1   1  1  Vậy:  x   x     x   x     x   x    4 1 2 2 3 3 1 x x x x x x .  1  2   2  3   3  1 
40 Nghiên cứu định lý Viète và ứng dụng | ▫▪ mathvn.com
October 4, 2021 [NGUYỄN THÀNH NHÂN – TRƯỜNG ĐẠI HỌC AN GIANG]
Dạng 13. So sánh nghiệm.
Định lý về dấu kết hợp định lý Viéte cho tam thức bậc hai:
S x x Đặt: 1 2  .
P x .x  1 2 .
a So sánh nghiệm với hằng số 0 .   0 '  0  
i Điều kiện để: x x  0 là: S  0   1 . 1 2 P  0    0 '  0  
ii Điều kiện để: 0  x x là: S  0 2. 1 2 P  0 
iii Điều kiện để: x  0  x là: P  0 3. 1 2 .
b So sánh nghiệm với hằng số  ,  (Với  ,  là các tham số thực cho trước).
i Điều kiện để: x   x là: .af   0 4 . 1 2    0 '  0  
ii Điều kiện để:   x x là:  . a f    0 5. 1 2 S     2    0 '  0  
iii Điều kiện để: x x   là:  . a f    0 6. 1 2 S     2
41 Nghiên cứu định lý Viète và ứng dụng | ▫▪ mathvn.com
October 4, 2021 [NGUYỄN THÀNH NHÂN – TRƯỜNG ĐẠI HỌC AN GIANG]   . a f     0
iv Điều kiện để: x      x là:  7. 1 2  . a f     0   . a f     0
v Điều kiện để: x    x   là:  8. 1 2  . a f     0 
x    x  
vi Điều kiện để: 1 2
là: f  . f    0 9. 
  x    x  1 2   0 '  0  .af     0  
vii Điều kiện để:   x x   là:  10 . 1 2 . a f      0   S      2
Để tiết kiệm thời gian cho một số bài toán ta có thể tính trực tiếp dựa trên hàm số bậc ba: 3 2
y ax bx cx d a  0 . Khi đó: a  0 
 Hàm số đồng biến trên khi và chi khi  . ' 2
  b  3ac  0  f ' a  0 
 Hàm số nghịch biến trên khi và chi khi  . ' 2
  b  3ac  0  f '
42 Nghiên cứu định lý Viète và ứng dụng | ▫▪ mathvn.com
October 4, 2021 [NGUYỄN THÀNH NHÂN – TRƯỜNG ĐẠI HỌC AN GIANG] Extra Techniques Study tips
Cơ sở hình thành các điều kiện:
 1 :Phương trình có hai nghiệm phân biệt nên   0 hoặc '  0. Do tổng của hai số âm
là một số âm và tích của hai số âm là một số dương.
2:Phương trình có hai nghiệm phân biệt nên   0 hoặc '  0. Do tổng của hai số
dương là một số dương và tích của hai số dương là một số dương. c
3 :Sỡ dĩ chỉ cần một điều kiện P  0 là vì khi P  0  P   0 hay a
ac  0  ac  0 . Khi đó: 2
   b  4ac  0 . Đã thỏa mãn điều kiện cần và đủ để
phương trình có hai nghiệm thực phân biệt.
4,7,8,9: gx trong khoảng hai nghiệm thì trái dấu với hệ số a và ở ngoài hai
khoảng nghiệm thì cùng dấu với hệ số a . Và tích của một số dương với một số âm là một số âm.  S
5 : Vì x x   nên x x  2x  2    6 : Vì x x   nên 2 1 1 2 1 2 1 2 S
x x  2x  2 
  6 : Vì   x x   nên 2x x x  2x . Khi đó: 1 2 2 2 1 2 1 1 2 2 S
2  S  2      . 2
Lưu ý: Ở các điều kiện5,6, 10 thì dấu của các tích . a f  , .
a f   đều được xét
dấu tương tự với 4, 7,8,9 . Và vì phương trình có hai nghiệm phân biệt nên   0 hoặc  '  0 . 2
Câu 1. Tìm m để hàm số 3 y  
x  m   2
1 x  2mx  5 đồng biến trên khoảng 0;2 . 3 Giải
43 Nghiên cứu định lý Viète và ứng dụng | ▫▪ mathvn.com
October 4, 2021 [NGUYỄN THÀNH NHÂN – TRƯỜNG ĐẠI HỌC AN GIANG] Ta có: 2 y '  2
x  2m   1 x  2m .
Để hàm số đồng biến trên khoảng 0;2 thì phương trình y '  0 có hai nghiệm thực phân
biệt x x thỏa mãn: x  0  2  x . Điều đó tương đương với hệ: 1 2 1 2  . a f  0  0  2.2  m  0 m  0      m  0.   . a f  2  0  2  m 12  0 m  6
Câu 2. Tìm tất cả các giá trị thực của tham số m để hàm số: 1 3
y   x  m   2
1 x  m  3 x 10 đồng biến trong khoảng 0;3 ? 3 Giải Ta có: 2
y '  x  2m  
1 x m  3 .
Để hàm số đồng biến trên 0;3 thì phương trình: y '  0 có hai nghiệm phân biệt x , x 1 2
thỏa mãn: x  0  3  x . Khi đó: 1 2
  m  2 2 ' 1  m  3  0
'  m m  4  0    1 . f 0  0   f 0  0      1 . f 3  0 f   3  0   m    m     12
 m  3  0  m  3   m  7   7m 12  0 12  m   7
Dạng 14. Ứng dụng khác.
Câu 1. Cho parabol  P 2
: y  x và đường thẳng d  đi qua điểm I 0;   1 và có hệ số
góc là k . Gọi A B là các giao điểm của  P và d . Giả sử ,
A B lần lượt có hoành độ là 
. Giá trị nhỏ nhất của biểu thức: 3 3 x x là? 1 2 Giải
44 Nghiên cứu định lý Viète và ứng dụng | ▫▪ mathvn.com
October 4, 2021 [NGUYỄN THÀNH NHÂN – TRƯỜNG ĐẠI HỌC AN GIANG] Cho parabol 2
y  x và đường thẳng d  đi qua điểm I 0;  
1 và có hệ số góc là k .
Gọi A B là các giao điểm của  P và d . Giả sử A B lần lượt có hoành độ là x , x . 1 2
+ Đường thẳng d  có phương trình: y kx 1.
+ Phương trình tương giao d  và P: 2 2
x kx 1  x kx 1  0   * .
+ * luôn có 2 nghiệm phân biệt: x ; x , vì: 2
  k  4  0, x   . 1 2
Theo định lý Viéte, ta có: x x  k, x x  1. 1 2 1 2 Ta có: 3 3 2
x x  (x x ) (x x )  x x
x x . (x x )  x x . 1 2 1 2  1 2 1 2  = 2 1 2 1 2 1 2 2 2 Ta có: x x
 x x  2
 4x x k  4 . 1 2 1 2 1 2 3 3  x x = 2 k   2 4. k   1  4.1  2 , k
  . Đẳng thức xảy ra khi k  0 . 1 2
Câu 2. Tìm tất các các giá trị thực m để đường thẳng y  x m cắt đồ thị hàm số 1 3 y
x  2  m 2
x  32m  3 x m tại ba điểm phân biệt A0;m, ,
B C sao cho đường 3
thẳng OA là phân giác của góc BOC . Giải
Phương trình hoành độ giao điểm: x  0 1 3 
x  2  m 2
x  32m  3 x m  x m  1 . 2 3
x  2  mx  6m 8  0 * 3
Để đường thẳng cắt đồ thị tại ba điểm phân biệt thì * phải có hai nghiệm phân biệt khác x  0 Hay:
45 Nghiên cứu định lý Viète và ứng dụng | ▫▪ mathvn.com
October 4, 2021 [NGUYỄN THÀNH NHÂN – TRƯỜNG ĐẠI HỌC AN GIANG]  18  8 3 m   44 3          m 4   m   2 2 m 12m 0 2 6 8  0  3  18  8 3  3    m    1 . 4      3 6m 8 0 m   3  4 m   3
Suy ra: Tọa độ các điểm B, C lần lượt là: B x , x m ,C x , x m . 1 1   2 2 
x x  3 m  2  1 2  
Theo định lý Viéte, ta có:  .
x x  3 6m  8  1 2   O  0;0Oy
Để ý: OA Oy (Do 
) có véctơ chỉ phương j 0;  1 . A
 0;mOy
Vậy để đường thẳng OA là phân giác của góc BOC . m x m x
cos  j,OB  cos j,OC 1 2   2. 2 2 2 2
x  (m x )
x  (m x ) 1 1 2 2
2  m x x  m x 2  m x x  m x 2 2 2 1 2 2 2 1 1
 m x 2 x  m x 2   m x 2 x  m x 2 2 2  1 2 2 2 1 1    
 m x 2 x m x . m xm x
x m x m x 2  2  2  2 2 2 2 1 1 1  2 2  1 2 2
x (m x )  x m x 2 2 2 2 2 2 2 2 2 2 2 2 2 2
x m  2mx x x .x x m  2mx x x x 2 1 1 2 2 1 2 1 2 1 1 2 1 2 2  m  2 2
x x   2mx x x x  2
 0  m x x x x
 2mx x x x  0 1 2 1 2 1 2  1 2  1 2  1 2  1 2   
mx x m
  x x mx mx 1 2  2x x   0  1 2 1 2 1 2 
m(x x )  2x x  . 1 2 1 2 m  0 m  0 m  0       m  7  33  2
3m(m  2)  6(6m  8)
3m  42m  48 m  7  33 
Đối chiếu điều kiện  
1 và A  O0;0 nên ta nhận m  7  33 .
46 Nghiên cứu định lý Viète và ứng dụng | ▫▪ mathvn.com
October 4, 2021 [NGUYỄN THÀNH NHÂN – TRƯỜNG ĐẠI HỌC AN GIANG]
Câu 3. Biết đồ thị hàm số y   x   x   2 1
1 x  7  m cắt trục hoành tại 4 điểm phân
biệt có hoành độ là x , x , x , x . Tìm tất cả các giá trị nguyên của tham số m để 1 2 3 4 1 1 1 1     1? 1 x 1 x 1 x 1 x 1 2 3 4 Giải
Xét phương trình hoành độ giao điểm:
x  x   2x   4 2 1 1
7  m  0  x  8x  7  m  0   1 Đặt: 2
t x ,t  0 . Khi đó phương trình đã cho có dạng: 2
t  8t  7  m  0 2
Vì đồ thị cắt trục hoành tại 4 điểm phân biệt nên phương trình   1 có 4 nghiệm thực
phân biệt hay phương trình 2 có hai nghiệm phân biệt dương. Điều đó tương đương '  0 16   7  m  0   với: S  0  8   0  9   m  7 .   P  0 7  m  0   t   t  8
Khi đó phương trình 2 có hai nghiệm dương: 0  t t thoả mãn: 1 2  . 1 2 t t  7  m  1 2 Suy ra phương trình  
1 có bốn nghiệm lần lượt là:
x   t , x   t , x
t , x t . 1 2 2 1 3 1 4 2 Như vậ 1 1 1 1 1 1 1 1 y:     1      1. 1  x 1  x 1  x 1  x 1  t 1  t 1  t 1  t 1 2 3 4 2 1 1 2 1 1 1 1 2 2
4  2t t 1 2       1    1   1 1  t 1  t 1  t 1  t 1  t 1  t 1  t tt t 2 2 1 1 2 1  1 2 1 2 4  2.8 12 12  m   1  1 
 0  0  m  12 1  8  7  m m m
Kết hợp điều kiện ta được: 0  m  7  m  1;2;3;4;5;  6 .
47 Nghiên cứu định lý Viète và ứng dụng | ▫▪ mathvn.com
October 4, 2021 [NGUYỄN THÀNH NHÂN – TRƯỜNG ĐẠI HỌC AN GIANG] Câu 4. Cho hàm số 3
y x    m 2 3 2
x  9m  3 
1 x  27  7m có đồ thị là C . Biết
rằng ứng với giá trị nguyên m m thì hàm số C cắt trục hoành tại 3 điểm lập thành 1
một cấp số cộng có các phần tử đều nguyên dương và ứng với giá trị nguyên m m thì 2
hàm số cắt trục hoành tại 3 điểm lập thành một cấp số nhân có các phần tử đều nguyên
dương. Tìm giá trị m , m ? 1 2 Giải Từ hàm số: 3
y x    m 2 3 2
x  9m  3 
1 x  27  7m .
Ta nhận thấy x  1 luôn luôn là một nghiệm của phương trình y  0 .
Bằng phép chia Hoocner ta có: y   x   2
1 x  4  2mx  7m  27 . x  1
Giao điểm của hàm số với trục hoành là nghiệm của:  . 2 x  
4  2mx  7m  27  0
Để hàm số cắt trục hoành tại 3 điểm phân biệt thì: '  
2  m2  7m  27  0 22   m  . 
 2    m 5 1 4 2 .1  7m  27  0
Giả sử 3 giao điểm của y f x với trục hoành lần lượt là: 1, x , x . 1 2 Khi đó:
- 3 điểm trên lập thành một cấp số cộng khi và chỉ khi:
2  3d  2m  4 x  1 d
x x  2m  4
2  3d  2m  4  1 1 2         3m  2 2 x  1 2d
x .x  7m  27  
2d  3d 1  7m  27
2d  3d 1  7  3  27 2 1 2      2   3d m   3 
2  3d  2m  4 2
d;m  2;6 TM       21  d  2     2 d d   d    d m 7 45 2 3 1 6 ;  ;   L  2 7     4 8  d   4 Hay: m  6 . 1
- 3 điểm trên lập thành một cấp số nhân khi và chỉ khi:
48 Nghiên cứu định lý Viète và ứng dụng | ▫▪ mathvn.com
October 4, 2021 [NGUYỄN THÀNH NHÂN – TRƯỜNG ĐẠI HỌC AN GIANG] 2
d d  2m  4 2 x d           1 x x 2m 4 d d 2m 4 1 2 2       
d d  4  . 2 3 3 x d
x .x  7m  27  
d  7m  27 d  7     27 2 1 2 2    2  d d  4 m   2
d d  2m  4 2       d  2
 d;m  2;5 7 7  TM  3 2 d d d 13  0    2 2 3  113 d  L   4 Nên: m  5 . 2 1 3 Câu 5. Cho hàm số 2 y
x mx  m  6 x  2021. Tìm tất cả các giá trị nguyên của 3
m thuộc để đồ thị hàm số có 5 điểm cực trị. Giải Đồ 1 3 thị hàm số 2 y
x mx  m  6 x  2021 có 5 điểm cực trị khi và chỉ khi đồ thị 3 1 hàm số 3 2 y
x mx  m  6 x  2021 có hai điểm cực trị nằm bên phải trục Oy hay 3 1 hàm số 3 2 y
x mx  m  6 x  2021 có hai điểm cực trị dương. 3 Ta có: 2
y  x  2mx m  6 .
Bài toán đã cho trở thành việc tìm m để phương trình 2
x  2mx  m  6  0 có hai
nghiệm dương phân biệt. Khi đó:  m  3   0  2
m m  6  0  m  2   b  
  0  2m  0
 m  0  m  3. a    m  6  0 m  6   c    0   a
49 Nghiên cứu định lý Viète và ứng dụng | ▫▪ mathvn.com
October 4, 2021 [NGUYỄN THÀNH NHÂN – TRƯỜNG ĐẠI HỌC AN GIANG] Câu 6. Cho hàm số 3 2
y ax bx cx d a, , b c, d
có đồ thị là đường cong như
hình vẽ. Định dấu của các hệ số , a , b , c d . Giải
Ta có: lim y    a  0 . x
Gọi x , x là hoành độ hai điểm cực trị của hàm số suy ra x , x nghiệm phương trình 1 2 1 2 2
y  3ax  2bx c  0 nên theo định lí Viéte, ta có: 2  b b
+) Tổng hai nghiệm: x x
 0   0  b  0 . 1 2 3a a c
+) Tích hai nghiệm: x .x   0  c  0 . 1 2 3a
Lại có đồ thị hàm số cắt trục tung tại điểm có tung độ dương nên d  0 . 2 4x x 12
Câu 7. Cho hàm số y
có đồ thị C . Tìm tập hợp S chứa tất cả các giá 2
x  6x  2k
trị thực của thám số k để đồ thị C có đúng hai tiệm cận đứng? Giải 0  x  4 Điều kiện:  . 2
x  6x  2k  0 Ta có: 2
12  4x x  0, x   D .
Nên để C có hai tiệm cận đứng thì phương trình: 2 2
x  6x  2k  0  x  6x  2k  0   *
có hai nghiệm phân biệt thuộc 0;4.
50 Nghiên cứu định lý Viète và ứng dụng | ▫▪ mathvn.com
October 4, 2021 [NGUYỄN THÀNH NHÂN – TRƯỜNG ĐẠI HỌC AN GIANG] Để 9
phương trình * có hai nghiệm phân biệt thì:   9  2k  0  k  . 2
Gọi hai nghiệm phân biệt của * là x x , ta có: 0  x x  4 . 1 2 1 2
Theo định lý Viéte ta có:  x x  0  2k  0 1 2   x x  6
x x  0  6  0  k  0 1 2 1 2         k  4 x x  2k x  4 x  4  0 2k  24 16  0    2k  8  0 1 2  1  2  
x  4  x  4  0    6  8  0 1   2   9 
Kết hợp nghiệm ta có: S  4;  .  2  
Câu 8. Với giá trị nào của tham số m thì phương trình x x 1 4  m2  2m  0 có hai
nghiệm x , x thỏa mãn x x  3 ? 1 2 1 2 Giải 2  Ta có: x x 1 4  2
 2  0  2x   2 .2x m m m  2m  0 * .
Xem phương trình * là phương trình bậc hai theo ẩn 2x t   0 . Khi đó:   2
*  t  2mt  2m  0 **.
Điều kiện để phương trình  
** có hai nghiệm dương phân biệt: 2
'  m  2m  0 
S  2m  0  m  2 .
P  2m  0 
Theo định lý Viéte, ta có: t .t  8  2m  8  m  4 . 1 2 Note:
Để phương trình * có hai nghiệm x , x thỏa mãn yêu cầu đề bài thì   ** phải có hai 1 2 
nghiệm dương phân biệt thỏa mãn t .t  8 do 1 x 2 x 3 1 x 2   3  2  2  8  2 .2x x x  8. 1 2 1 2 2 x 1 0
Câu 8. Xác định m để bất phương trình sau có nghiệm:  . 2 x  2  m  
1 x  4m 1  0 Giải 2 x 1 0    1 Đặt:  . 2 x  2  m  
1 x  4m 1  0 2
51 Nghiên cứu định lý Viète và ứng dụng | ▫▪ mathvn.com
October 4, 2021 [NGUYỄN THÀNH NHÂN – TRƯỜNG ĐẠI HỌC AN GIANG]  1   x 1  Từ   1   . 2 
   m  2m 2
+) Trường hợp 1:   0  hệ phương trình đã cho có nghiệm đúng với mọi x 1  ;  1 2 2
m  2m  0  m0;2   * . m  0
+) Trường hợp 2:   0 
 hệ phương trình có nghiệm  2 có nghiệm 2  m  2 m  0 m  0        m  2 m 2  0 2    2     f   1  6m  4  0     . 1 0 m a f       m   3 2 1  2 S            m  2 1 m 0  1   3 x x  1   2     2 1 2          m  2  * . x x  1      1 2 m 0        m  0 m  0   0  2    m  2         a f   m 2 . 1  0     f   1  2m  0    m  0 S   1 2   m   1    m  0 2  1   2    2 Từ   * *  *  m   . 3
Câu 9. Cho các số thực a, b, c (với a  0) sao cho phương trình 2
ax bx c  0 có hai
(a b)(2a b)
nghiệm thuộc đoạn 0 
;1 . Tìm giá trị lớn nhất của biểu thức: P  .
a(a b c) Giải  b x x    1 2  a
Gọi x , x là nghiệm của phương trình đã cho. Theo định lý Viéte, ta có:  . 1 2 cx x  1 2  a
52 Nghiên cứu định lý Viète và ứng dụng | ▫▪ mathvn.com
October 4, 2021 [NGUYỄN THÀNH NHÂN – TRƯỜNG ĐẠI HỌC AN GIANG] Do a  0 , nên:  b   b  1 2      a  a
1 x x 2 x x 2 1 x x  1 x x x x 1 2   1 2   1 2   1 2   1 2  P     b c
1 x x x x
1 x x x x 1 2 1 2 1 2 1 2 1     a a
2 1 x x    x x    x x 2
2 1 x x x x    2 2
x x x x  2 2    1 2 1 2 1 2 1 2 x x x x 1 2 1 2 1 2 1 2 1 2  P    2 
1 x x x x
1 x x x x
1 x x x x 1 2 1 2 1 2 1 2 1 2 1 2
Giả sử x x do 2 nghiệm thuộc 0  ;1 nên 2 2
x x x x  1 . 1 2 1 1 2 2 2 2
x x x x
x x 1 x x
Và 1 x x x x  0 nên ta có: 1 2 1 2 1 2 1 2  1 P  3 . 1 2 1 2
1 x x x x
1 x x x x 1 2 1 2 1 2 1 2 Vậy max P  3 .
Dấu đẳng thức xảy ra khi và chỉ khi: x  0 1 c  0  2  x x x x  1         1 1 2 2 b a 0 2  x 1  x 1  2 1 b 
a c    0  x 1   2 2  2m 1 Câu 10. Cho hàm số 3 2 y x
x  2x  2022  
. Biết rằng tồn tại hai giá trị tham số  2 
m m ; thỏa mãn hàm số đạt cực trị tại hai điểm x , x sao cho 2x  3x  3  m . Tìm m , m . 1 2 1 2 1 2 1 2 Giải  2m 1 3 2 2 y x
x  2x  2022; y '  3x    2m   1 x  2  2 
Hàm số có hai điểm cực trị khi phương trình y '  0 có hai nghiệm phân biệt 1
  2m  2 1
 6  0  2m  2 1  24,(*) 4
53 Nghiên cứu định lý Viète và ứng dụng | ▫▪ mathvn.com
October 4, 2021 [NGUYỄN THÀNH NHÂN – TRƯỜNG ĐẠI HỌC AN GIANG]  2m 1 x x   1 2  Theo đị 3 nh lý Viète ta có:  . 2  x x  1 2  3   1 2m 1 x  3m  2     1   x x  5 Vì: 1 2  3   . 1
2x 3x 3  m x m 11 1 2 2     15 2
Thế vào x .x  , ta được: 1 2 3 2 1    x x   3m  2 1 . m   2 1 11        2 2
3m  31m  22  . 1 2  3 5  15  3 75 3  1 m  5 73  31  1    2 2 6
3m  31m  22  50  3m  31m  72  0   . 1
m   5 73  31 2    6
Câu 11. Xác định giá trị của tham số m để hàm số
y f x  m   3
x  m   2 1 3
1 x  2mx  4 đồng biến trên khoảng có độ dài bằng 1 . Giải Ta xét các khả năng:
Trường hợp 1: m  1
 , khi đó: y f x  2
x  4 là một đường thẳng có hệ số góc k  2
  0 . Nên hàm số nghịch biến trên .
Trường hợp 2: m  1  , khi đó:
Nếu f ' x  0 vô nghiệm hoặc có nghiệm kép thì hàm số luôn đồng biến trên . (Loại)
Nếu f ' x  0 có hai nghiệm thực x , x thì hàm số đồng biến trên hai khoảng  ;  x 1  1 2
và  x ; . (Loại) 2 
Như vậy ở Trường hợp 2 không tồn tại giá trị thực m nào để cho hàm số đồng biến trên
khoảng có độ dài bằng 1 .
Trường hợp 3: m  1  , khi đó:
54 Nghiên cứu định lý Viète và ứng dụng | ▫▪ mathvn.com
October 4, 2021 [NGUYỄN THÀNH NHÂN – TRƯỜNG ĐẠI HỌC AN GIANG]
Nếu f ' x  0 vô nghiệm hoặc có nghiệm kép thì hàm số luôn nghịch biến trên . (Loại)
Nếu f ' x  0 có hai nghiệm thực x , x thì hàm số đồng biến trên khoảng  x ; x . 1 2  1 2 m  1  
Như vậy để hàm số đồng biến trên khoảng có độ lớn bằng 1 thì: '  . f  0 * ' x   
x x  1  1 2
Ta có: f x  m   2 ' 3
1 x  6m   1 x  2m .    2 m 1 Suy ra:  '  m
m m   m m    . f  9 1 6 1 1 3 9 0 ' x         m  3  x x  2 1 2 
Theo định lý Viète, ta có:  2m . x x  1 2  3m   1 2 2
x x  1  x x
 1  x x  4x x  1 1 2  1 2  1 2 1 2  m  1   m  3     m  1   *      2mm  9  . 2 m  3  2  4.   3m    1 1  2 2m 2  4.    m   1 3 1 ----The End----
55 Nghiên cứu định lý Viète và ứng dụng | ▫▪ mathvn.com
October 4, 2021 [NGUYỄN THÀNH NHÂN – TRƯỜNG ĐẠI HỌC AN GIANG]
56 Nghiên cứu định lý Viète và ứng dụng | ▫▪ mathvn.com